Tải bản đầy đủ (.doc) (64 trang)

Dai so 9C4

Bạn đang xem bản rút gọn của tài liệu. Xem và tải ngay bản đầy đủ của tài liệu tại đây (443.5 KB, 64 trang )

<span class='text_page_counter'>(1)</span><div class='page_container' data-page=1>

<i>Ngày </i>


<i>soạn:</i>

17/01/2010



<i>Ngày dạy:</i>

<sub>18/01/2010</sub>



Tiết: 47


Chương IV HÀM SỐ Y = A X<b>2</b>

<b><sub> (A </sub></b>



<b> 0)</b>


<b>PHƯƠNG TRÌNH BẬC HAI MỘT ẨN</b>


§1 . Hàm số y = a x<b>2</b>

<b><sub> ( a </sub></b>



<b> 0)</b>



I. <b>Mục tiêu</b>


HS nắm được:


- Trong thực tế có những hàm số dạng y = ax2<sub> ( a </sub><sub></sub><sub> 0 ).</sub>


- Tính chất và nhận xét về hàm số y = ax2<sub> ( a </sub><sub></sub><sub> 0).</sub>


- HS biết cách tính giá trị của hàm số tương ứng với giá trị cho trước của biến số.
- Về tính thực tiễn: HS thấy được thêm một lần nữa mối liên hệ hai chiều của Toán
học với thực tế: Toán học xuất phát từ thực tế và nó quay trở lại phục vụ thực tế.
II. <b>Chuẩn bị</b>


GV: Bảng phụ ghi ?1,?4; MTBT.


HS: MTBT.


<b>III. Tiến trình dạy - học</b>


<b>Hoạt động 1</b> (5’)<b> Đặt vấn đề và giới thiệu chương</b>


GV: ở chương II, chúng ta đã nghiên cứu hàm số bậc nhất và đã biết rằng nó nảy
sinh từ những nhu cầu của thực tế cuộc sống. Nhưng trong thực tế cuộc sống, ta thấy
có nhiều mối liên hệ được biểu thị bởi hàm số bậc hai. Và cũng như hàm số bậc nhất,
hàm số bậc hai quay trở lại phục vụ thực tế như giải phương trình, giải tốn bằng cách
lập phương trình hay một số bài tốn cực trị. Tiết học này và tiết học sau, chúng ta sẽ
tìm hiểu tính chất và đồ thị của một dạng hàm số bậc hai đơn giản nhất. Bây giờ, ta
hãy xem một ví dụ.


<b>Hoạt động 2</b> (8’)


GV: Nhìn vào bảng trên, em hãy
cho biết


s1 = 5 được tính như thế nào?


s4 = 80 được tính như thế nào?


GV: Trong cơng thức s = 5t2<sub>, </sub>


nếu thay s bởi y, thay t bởi x,
thay 5 bởi a thì ta có cơng thức
nào?


GV: Trong thực tế còn nhiều cặp


đại lượng cũng liên hệ bởi công
thức dạng y = ax2<sub>( a </sub><sub></sub><sub> 0) như </sub>


diện tích hình vng và cạnh của
nó ( S = a2<sub>) , diện tích hình trịn </sub>


và bán kính của nó ( S = R2)…


Hàm số y = a x2 <sub> ( a </sub>


 0 ) là


dạng đơn giản nhất của hàm số
bậc hai. Sau đây chúng ta sẽ xét
các tính chất của nó.


<b>1. Ví dụ mở đầu</b>


( SGK)


Quãng đường chuyển động của vật rơi tự do được
biểu diễn gần đúng bằng công thức s = 5 t2<sub> </sub>


Ta có:


t 1 2 3 4


s 5 20 45 80


y = ax2<sub> ( a </sub>


 0)


</div>
<span class='text_page_counter'>(2)</span><div class='page_container' data-page=2>

<b>Hoạt động 3</b> (20’)
HS làm ?1


GV đưa bảng phụ ?1
GV: Gọi 2 HS lên bảng
HS làm ?2.


GV: Đối với hai hàm số cụ thể là
y = 2x2 <sub>và y = -2x</sub>2<sub> thì ta có các </sub>


kết luận trên. Tổng quát, người
ta chứng minh được hàm số y =
ax2<sub> ( a </sub>


 0) có tính chất


HS đọc tính chất (SGK).


HS làm ?3 theo nhóm.


GV: Gọi đại diện nhóm trình bày
kết qủa.


GV nêu nhận xét SGK


<b>2. Tính chất của hàm số y = a x2</b><sub> ( a </sub>


 0)



Xét hai hàm số: y = 2x2<sub> và y = -2x</sub>2


?1. Điền vào chỗ trống các giá tri tương ứng của y
trong hai bảng sau:


x -3 -2 -1 0 1 2 3


y = 2x2 <sub>18</sub> <b><sub>8</sub></b> <b><sub>2</sub></b> <b><sub>0</sub></b> <b><sub>2</sub></b> <sub>8</sub> <b><sub>18</sub></b>


y = -2x2 <sub>-18</sub> <b><sub>-8</sub></b> <b><sub>-2 0</sub></b> <b><sub>-2</sub></b> <sub>-8</sub> <b><sub>-18</sub></b>


?2. Đáp


* Đối với hàm số y = 2x2


- Khi x tăng nhưng luôn âm thì y giảm.
- Khi x tăng nhưng ln dương thì y tăng.
* Đối với hàm số y = -2x2<sub>.</sub>


- Khi x tăng nhưng ln âm thì y tăng.
- Khi x tăng nhưng ln dương thì y giảm.


Tổng qt : Hàm số y = ax2<sub> ( a </sub>


 0 ) xác định với


mọi giá trị của x thuộc R, có tính chất sau:
<i> Nếu a > 0 thì hàm số nghịch biến khi x < 0 và </i>
<i>đồng biến khi x> 0.</i>



<i> Nếu a < 0 thì hàm số đồng biến khi x< 0 và </i>
<i>nghịch biến khi x > 0.</i>


?3. Đáp


- Đối với hàm số y = 2x2<sub>, khi x </sub><sub></sub><sub> 0 thì giá trị của y</sub>


ln dương, khi x = 0 thì y = 0.
- Đối với hàm số y = -2x2<sub>, khi x </sub>


 0 thì giá trị của


hàm số ln âm, khi x = 0 thì y = 0.


<i><b>Nhận xét </b></i>


Nếu a > 0 thì y > 0 với mọi x  0 ; y = 0 khi x=


</div>
<span class='text_page_counter'>(3)</span><div class='page_container' data-page=3>

HS làm ?4. (bảng phụ)


HS1: Điền các giá trị bảng y =


2
1


x2


HS2: Điền các giá trị bảng y



=-2
1


x2


0.


Giá trị nhỏ nhất của hàm số là y = 0.


Nếu a < 0 thì y < 0 với mọi x  0 ; y = 0 khi x =


0.


Giá trị lớn nhất của hàm số là y = 0.
?4.Đáp số.


x -3 -2 -1 0 1 2 3


y = 1<sub>2</sub> x <b>4</b>1<sub>2</sub> <b>2</b> <sub>2</sub>1 <b>0</b> <sub>2</sub>1 <b>2</b> <b>4</b><sub>2</sub>1


y = -1<sub>2</sub>
x


<b>-4</b> <sub>2</sub>1 <b>-2</b> <b>-</b>1<sub>2</sub> <b>0</b> <b></b>


-2
1


<b>-2 -4</b>1<sub>2</sub>



<b>Hoạt động 4</b> (10’)
HS làm bài 1 SGK.


HS sử dụng MTBT để làm bài
tập 1a ( SGK)


HS trả lời miệng câu b,c


<b>Luyện tập:</b>


Bài 1.a, dùng MTBT để tính các giá trị của S rồi
điền vào ô trống.


R( cm) 0,57 1,37 2,15 4,09


S = R2 ( cm) <b>1,02</b> <b>5,89</b> <b>14,52 52,53</b>


b, Nếu bán kính tăng gấp 3 lần thì diện tích tăng : 9
lần


c, S = 79,5 cm2<sub> thì R = ?</sub>


Từ S = R2 R =


<i>S</i>


= 79<sub>3</sub><sub>,</sub><sub>14</sub>,5  5,03 ( cm)
<b>Hoạt động 5 </b> <b>Hướng dẫn về nhà</b> (2’)



- Học nắm chắc lí thuyết


- Làm bài tập 2,3 ( SGK); 1,2 ( SBT)


<i> Diễn Bích, ngày tháng năm 2010</i>
BGH kí duyệt


<i>Ngày soạn:</i>

<sub>19/01/2010</sub>

<i>Ngày dạy:</i> <i>20/01/2010</i>


Tiết:48 <sub>§ 2. </sub><b><sub>ĐỒ THỊ CỦA HÀM SỐ</sub></b>

<b><sub> y = ax</sub></b>

<b>2</b>

<b><sub> ( a </sub></b>

<sub></sub>

<b><sub> 0)</sub></b>



I. <b>Mục tiêu</b>


</div>
<span class='text_page_counter'>(4)</span><div class='page_container' data-page=4>

-HS biết được dạng của đồ thị hàm số y = ax2<sub> ( a </sub><sub></sub><sub> 0 ) và phân biệt được chúng trong</sub>


hai trường hợp a> 0; a < 0.


- Nắm vững tính chất của đồ thị và liên hệ được tính chất của đồ thị với tính chất của
hàm số.


- Biết cách vẽ đồ thị y = ax2<sub> ( a </sub>
 0 ).


II. <b>Chuẩn bị</b>


GV: Bảng phụ vẽ sẵn bảng giá trị của hàm số y = 2x2 <sub>và y = - </sub>


2
1



x2<sub>.</sub>


vẽ sẵn đồ thị các hàm số đó


HS : Ơn lại các kiến thức “Đồ thị hàm số y = f(x)”, cách xác định một điểm của đồ thị
- Thước kẻ và MTBT.


<b>III. Tiến trình dạy - học</b>
<b>Hoạt động 1</b> (8’) <b>Kiểm tra:</b>


GV đưa bảng phụ, y/c hai HS lên bảng


HS1: Điền vào ô trống các giá trị tương ứng của y trong bảng sau:


x -3 -2 -1 0 1 2 3


y = 2x2


- Hãy nêu tính chất của hàm số y = ax2<sub> ( a </sub>
 0)


HS2: Hãy điền vào những ô trống các giá trị tương ứng của y trong bảng sau:


x - 4 -2 -1 0 1 2 4


y = -<sub>2</sub>1
x2


- Hãy nêu nhận xét sau khi học hàm số y = ax2<sub> ( a </sub><sub></sub><sub> 0)</sub>



<b>Hoạt động 2</b> (20’)


GVđvđ: Ta đã biết đồ thị của hàm số
y = ax+ b ( a  0 ) có dạng là một


đường thẳng, tiết học này ta sẽ xét
xem đồ thị của hàm số y = a x2<sub>(a </sub>




0) có dạng như thế nào?
Ta xét VD1.( Bảng phụ)


GV: Em có nhận xét gì về dạng của
đồ thị?


GV giới thiệu cho HS tên gọi của đồ
thị là Parabol.


HS làm ?1.


GV nêu VD 2.


HS lên bảng lấy các điểm trên mặt
phẳng toạ độ.


<b>1. Các ví dụ:</b>


VD1: Đồ thị hàm số y = 2x2<sub> ( a = 2 > 0)</sub>



x -3 -2 -1 0 1 2 3


y = 2x2 <sub>18</sub> <sub>8</sub> <sub>2 0 2 8 18</sub>


?1. HS trả lời miệng


- Đồ thị hàm số y = 2x2<sub> nằm phía trên trục </sub>


hồnh.


- A và A’ đối xứng với nhau qua trục Oy;
B và B’ đối xứng với nhau qua trục Oy;
C và C’ đối xứng với nhau qua trục Oy.
- Điểm O là điểm thấp nhất của đồ thị.
VD2.Vẽ đồ thị hàm số y = -<sub>2</sub>1 x2


x - 4 -2 -1 0 1 2 4


</div>
<span class='text_page_counter'>(5)</span><div class='page_container' data-page=5>

M(- 4;- 8), N(-2;-2), P(- 1;1<sub>2</sub> ),
O( 0 ; 0) , P’(1; -<sub>2</sub>1 ) ; N’( 2; - 2) ,
M’( 4; - 8) rồi lần lượt nối chúng để
được một đường cong.


HS làm ?2.


GV: Từ 2 VD trên, em có nhận xét gì
về dạng và vị trí của đồ thị ttrong
từng trường hợp a > 0, a < 0?
GV : Nêu nhận xét.



<b>Hoạt động 3:</b>


HS đọc nhận xét ( SGK)


HS làm ?3.


GV: Muốn tìm điểm D có hồnh độ
bằng 3 ta làm như thế nào?


GV: Cách xác định tung độ điểm D
bằng đồ thị?


GV: Ta tính y như thế nào?


GV: Nếu khơng u cầu tính điểm D
bằng hai cách thì em chọn cách nào?
vì sao?


HS: Chọn cách 2, vì độ chính xác
cao hơn.


HS : Hồnh độ của điểm E’  3,16.


GV: Hãy kiểm tra lại bằng tính tốn.
GV u cầu HS dựa vào nhận xét
trên, hãy điền số thích hợp vào ơ
trống mà khơng cần tính tốn.
HS đọc chú ý ( SGK).


GV nhấn mạnh:



Khi vẽ đồ thị hàm số y = ax2<sub> ( a </sub><sub></sub><sub> 0 )</sub>


y = -<sub>2</sub>1 x2 -8 -2 1


2


 0 1


2


 -2 -8


<b>2. Nhận xét :</b>


- Đồ thị của hàm số y = - <sub>2</sub>1 x2<sub> nằm phía dưới </sub>


trục hồnh.


- M và M’ đối xứng nhau qua trục Oy,
N và N’ đối xứng nhau qua trục Oy,
P và P’ đối xứng nhau qua trục Oy.
- Điểm O là điểm cao nhất của đồ thị.
* Nhận xét: ( SGK)


?3.


a, Trên đồ thị, xác định điểm D có hoành độ 3.
- Bằng đồ thị suy ra tung độ của điểm D
bằng - 4,5.



- Tính y với x = 3, ta có:
y = -1<sub>2</sub> x2<sub> = </sub>


-2
1


. 32<sub> = -4,5.</sub>


Hai kết quả bằng nhau.


b, Trên đồ thị, điểm E và E’ đều có tung độ
bằng -5.


Giá trị hoành độ khoảng -3,2 và khoảng 3,2.
Điền vào chỗ trống.


x -3 -2 -1 0 1 2 3


y = <sub>3</sub>1 x2 3


3
4


3


1 <sub>0</sub>


3
1



3


4 <sub>3</sub>


* Chú ý ( SGK)


<b>Hoạt động 4</b> ( )
HS làm bài 4 SGK


GV đưa bảng lên bảng phụ và lưới ô
vuông để vữ đồ thị.


<b>Luyện tập: </b>


Điền vào ô trống của các bảng sau:


x -2 -1 0 1 2


</div>
<span class='text_page_counter'>(6)</span><div class='page_container' data-page=6>

Y/c một HS lên điền vào hai bảng


GV gọi 2 HS lên bảng vẽ đồ thị hai
hàm số


2


3


y x



2


 6 3


2 0


3


2 6


2


3


y x


2


 - 6 - 3


2 0 -


3


2 - 6


<b>Nhận xét: </b>Đồ thị hai hàm số nhận trục Ox
làm trục đối xứng


<b>Hoạt động 5 </b> <b>Hướng dẫn về nhà</b> (2’)


- BTVN: 5, 6, 7, 8 ( SGK); 7, 8, 9 (SBT)
- Đọc bài đọc thêm: “ Vài cách vẽ Parabol”


<i> Diễn Bích, ngày tháng năm 2010</i>
BGH kí duyệt


<i>Ngày soạn:</i>

<sub>24/01/2010</sub>

<i>Ngày dạy:</i>

<sub>25/01/2010</sub>



Tiết: 49 <b>LUYỆN TẬP</b>


I. <b>Mục tiêu</b>


- Củng cố cho HS nhận xét về đồ thị hàm số y = ax2<sub> ( a </sub>
 0) .


119


1,5


-1 1


-2 2


7


-1,5


-7


x


y


O
6


</div>
<span class='text_page_counter'>(7)</span><div class='page_container' data-page=7>

- HS được rèn luyện kỹ năng vẽ đồ thị hàm số y = ax2<sub> ( a </sub><sub></sub><sub> 0) . Kỹ năng ước lượng </sub>


các giá trị hay ước lượng vị trí của một số điểm biểu diễn các số vô tỷ.


- HS được biết thêm mối quan hệ chặt chẽ của hàm số bậc nhất và hàm số bậc hai để
sau này có thêm cách tìm nghiệm phương trình bậc hai bằng đồ thị, cách tìm GTLN,
GTNN qua đồ thị.


II. <b>Chuẩn bị</b>


GV: Bảng phụ vẽ sẵn đồ thị hàm số của bài tập 6, 7,8.
HS: Thước kẻ, MTBT.


<b>III. Tiến trình dạy - học</b>
<b>Hoạt động 1</b> (10’’)


HS1: Hãy nêu nhận xét đồ thị của hàm
số y = ax2<sub> ( a </sub><sub></sub><sub> 0).</sub>


- Làm bài tập 6a, b.


- HS khác nhận xét.
- GV nhận xét - đánh giá.


<b>Kiểm tra:</b>



a, Vẽ đồ thị hàm số y = x2


x -2 -1 0 1 2


y =x2 <sub>4</sub> <sub>1</sub> <sub>0</sub> <sub>1</sub> <sub>4</sub>


b, f(-8) = 64; f(-1,3) =1,69.
f(- 0,75) = <sub>16</sub>9 f(1,5) = 2,25
Hoạt động 2 (33’)


GV đưa bảng phụ vẽ hình 10 và u
cầu HS thảo luận nhóm bài 7 ( SGK).
Mỗi nhóm làm một câu


GV: Hãy cho biết toạ độ của điểm M?
GV: Muốn tìm hệ số a ta làm như thế
nào?


<b>Luyện tập:</b>


Bài 7 ( SGK)
Giải.


a, Điểm M (2;1)  x = 2; y = 1.


Thay x = 2, y =1 vào y= a x2<sub> ta có:</sub>


1 = a.22



 a = <sub>4</sub>1 .


120


f x <sub> = x</sub>2


O


-2 -1 1 2
4


1


</div>
<span class='text_page_counter'>(8)</span><div class='page_container' data-page=8>

GV: Muốn xét xem điểm A(4;4) có
thuộc đồ thị khơng ta làm như thế nào?


GV: Hãy tìm thêm 2 điểm nữa ( không
kể điểm O) để vẽ đồ thị .


GV: Gọi đại diện 1 nhóm lên bảng làm
câu c.


GV nêu thêm câu hỏi.


( nội dung câu hỏi bài tập 8)


d, Em tìm tung độ của điểm thuộc
Parabol có hồnh độ x = -3 như thế
nào?



HS: Cách 1: Dùng đồ thị
Cách 2: Tính tốn.


e, Muốn tìm các điểm thuộc Parabol có
tung độ y = 6,25 ta làm như thế nào?
HS: Cách 1: Dùng đồ thị : Trên Oy ta
lấy điểm 6,25, qua đó kẻ 1 đường song
song với Ox cắt Parabol tại B, B’.
Cách 2: Tính toán.


g, Khi x tăng từ -2 đến 4, qua đồ thị
hàm số đã vẽ, giá trị nhỏ nhất và lớn
nhất của y là bao nhiêu?


( nội dung câu hỏi bài 10 - SGK)
HS làm bài 9 SGK.


GV: Để vẽ đồ thị của hia hàm số trên
thì ta phải làm gì ?


GV:Hãy lập bảng giá trị của hai hàm số


b, Từ câu a, ta có : y = <sub>4</sub>1 x2


Điểm A( 4; 4)  x = 4; y = 4.


Với x = 4 thì <sub>4</sub>1 x2<sub> = </sub>


4
1



. 42<sub> = 4 = y.</sub>


Vậy điểm A thuộc đồ thị hàm số y = <sub>4</sub>1 x2.


c, Lấy 2 điểm nữa (không kể điểm O)
thuộc đồ thị là M’( -2; 1) và A’( -4; 4)
- Vẽ đồ thị .


d, Với x = -3 thay vào ta có:
y = <sub>4</sub>1 x2<sub> = </sub>


4
9


= 2,25


e, Với y = 6,25 thay vào biểu thức y = <sub>4</sub>1 x2


ta có: 6,25 = 1<sub>4</sub> x2


 x2 = 25  x =  5.
 B( 5; 6,25) ; B’(-5; 6,25) là hai điểm cần


tìm.


g, Khi x tăng từ -2 đến 4, giá trị nhỏ nhất
của y = 0 khi x = 0, còn giá trị lớn nhất của
y = 4 khi x = 4.



Bài 9: (SGK)
Hàm số y =


3
1


x2


x -3 -2 -1 0 1 2 3


y=


3
1


x2 3


3
4


3


1 <sub>0</sub>


3
1


3


4 <sub>3</sub>



Hàm số y = -x +6


x 0 6


y = -x + 6 6 0


121


f x  = 1
4

 

x2


O
y


</div>
<span class='text_page_counter'>(9)</span><div class='page_container' data-page=9>

GV gọi 1 HS vẽ Parabol và 1 HS vẽ
đường thẳng trên cùng một mặt phẳng
toạ độ.


GV: Hãy tìm toạ độ giao điểm của hai


đồ thị ? b, Toạ độ giao điểm của hai đồ thị là:


A(3;3) ; B (- 6; 12)


<b>Hoạt động 3:</b> <b>Hướng dẫn về nhà</b> (2’)
- Làm bài 9,10 ( SBT)


- Đọc phần “ Có thể em chưa biết”.



<i> Diễn Bích, ngày tháng năm 2010</i>
BGH kí duyệt


<i>Ngày soạn:</i>

<sub>24/01/2010</sub>

<i>Ngày dạy:</i>

<sub>27/01/2010</sub>



Tiết: 50 <b>§3. PHƯƠNG TRÌNH BẬC HAI MỘT ẨN</b>


I. <b>Mục tiêu</b>


- HS nắm được định nghĩa phương trình bậc hai một ẩn dạng tổng quát, dạng đặc biệt
khi b hoặc c bằng 0 hoặc cả b và c bằng 0. Luôn chú ý nhớ a khác 0.


- HS biết phương pháp giải riêng các phương trình thuộc hai dạng đặc biệt, giải thành
thạo các phương trình thuộc hai dạng đặc biệt đó.


- HS biết biến đổi phương trình dạng tổng quát:
ax2<sub> + bx + c = 0 ( a </sub><sub></sub><sub> 0 ) về dạng ( x + </sub>


<i>a</i>
<i>b</i>


2 )


2<sub> = </sub>


2
2


4


4


<i>a</i>
<i>ac</i>


<i>b</i>  <sub> trong các trường hợp cụ </sub>


thể của a, b, c để giải phương trình.


- HS thấy được tính thực tế của phương trình bậc hai một ẩn.


<b>II. Chuẩn bị</b>


122


-6 -3 O 3 6 x
y


12


</div>
<span class='text_page_counter'>(10)</span><div class='page_container' data-page=10>

Giáo án Đại số 9 năm học 2010-2011
GV: Bảng phụ ghi VD1,VD3, ?1, ?4.
HS: Ôn cách giải phương trình


<b>III. Tiến trình dạy - học</b>
<b>Hoạt động 1</b> ( ’)


GV nêu bài toán.


GV: Bài tốn cho biết gì? u cầu làm gì?


GV : Để giải bài toán này, ta gọi bề rộng
mặt đường là x( m) . 0 < 2x < 24


GV: Chiều dài phần đất còn lại là bao
nhiêu? Chiều rộng phần đất cịn lại là
bao nhiêu?


GV: Diện tích hình chữ nhật cịn lại là
bao nhiêu?


GV: Dựa vào dự kiện của bài tốn , hãy
lập phương trình.


GV: Hãy biến đổi để được phương trình
đơn giản.


GV giới thiệu phương trình bậc hai một
ẩn số .


<b>Bài tốn mở đầu:</b>


( SGK)
Giải


Gọi bề rộng mặt đường là x( m) .
0 < 2x < 24


Chiều dài là 32 - 2x ( m)
Chiều rộng là 24 - 2x (m)



Diện tích là ( 32 - 2x) ( 24 - 2x) ( m2<sub>)</sub>


Theo bài ra ta có phương trình.
( 32 - 2x) ( 24 - 2x) = 560
hay x2<sub> - 28x + 52 = 0</sub>


Phương trình x2<sub> - 28x + 52 = 0 được gọi </sub>


là phương trình bậc hai một ẩn.


<b>Hoạt động 2</b> ( ’)


GV giới thiệu định nghĩa phương trình
bậc hai một ẩn.


HS đọc định nghĩa SGK.


GV: Hãy lấy ví dụ về phương trình bậc
hai một ẩn? chỉ rõ ẩn, các hệ số a, b,c?
HS làm ?1.


GV đưa bảng phụ


<b>Định nghĩa:</b>


Phương trình bậc hai một ẩn là phương
trình có dạng ax2<sub> + bx + c = 0, trong đó x</sub>


là ẩn ; a,b,c là những số cho trước gọi là
các hệ số và a  0.



VD: a, x2<sub> + 24x - 75 = 0 b, 5x</sub>2<sub> - 4x = 0 </sub>


?1. Đáp số


- Phương trình a,c,e là phương trình bậc
hai một ẩn.


- Phương trình b, d khơng phải là phương
trình bậc hai một ẩn.


<b>Hoạt động 3</b> ()
HS xem ví dụ 1 SGK


GV: Em có nhận xét gì về phương trình ở
ví dụ 1? ( khuyết c)


GV: Nêu cách giải ? (đặt nhân tử chung)
HS làm ?2


GV nêu VD2 và yêu cầu HS nêu cách
giải.


<b>3. Một số ví dụ về giải phương trình </b>
<b>bậc hai:</b>


<b>Ví dụ 1: </b>3x2<sub> – 6x = 0</sub>


<b>?</b>2. Giải .



2x2<sub>+ 5x = 0 </sub><sub></sub><sub> x ( 2x + 5) = 0</sub>


 x = 0 hoặc x = - <sub>2</sub>5


Vậy phương trình có hai nghiệm:
x1= 0; x2=- <sub>2</sub>


5




123


</div>
<span class='text_page_counter'>(11)</span><div class='page_container' data-page=11>

HS làm ?3.


GV bổ sung thêm phương trình.
x2<sub> + 1 = 0</sub>


HS có thể giải cách khác:
x2


 0  x2 + 1  0  x2 + 1 không thể


bằng 0  vế trái  vế phải với mọi x
 phương trình vơ nghiệm.


GV: Từ bài giải ở HS2, HS3 em có nhận
xét gì về số nghiệm của phương trình bậc
khuyết b?



HS: Phương trình bậc hai khuyết b có thể
có nghiệm ( là hai số đối nhau), có thể vơ
nghiệm


GV hướng dẫn HS làm ?4.


HS làm ?5.


GV: Em có nhận xét gì về vế trái của
phương trình?


( Đưa phương trình về PT ?4)
HS thảo luận nhóm ?6, ?7.


GV: Gọi đại diện 2 nhóm lên bảng.


?3. Giải. 3x2<sub> - 2 = 0 </sub>


 3x2 = 2


 x2 = <sub>3</sub>2


 x = 
3
2


= 
3


6



Vậy phương trình có hai nghiệm
x1 =


3
6 <sub>; x</sub>


2 =
-3


6


* x2<sub> + 1 = 0 </sub>


 x2 = -1


Phương trình vơ nghiệm vì vế phải là một
số âm, vế trái là một số không âm.


?4. Giải phương trình ( x-2)2<sub> = </sub>


2
7


( x-2)2<sub> = </sub>


2
7


 x - 2 =  <sub>2</sub>7



 x = 2  <sub>2</sub>7  x =
2


14
4
Vậy phương trình có hai nghiệm là:
x1 = 4 14


2


 <sub> ; x</sub>


2 = 4 14


2




?5.


?6. Giải phương trình. x2<sub> - 4x = - </sub>


2
1


Thêm 4 vào hai vế, ta có:


 x2 - 4x + 4 = -<sub>2</sub>1 + 4  (x - 2)2 = <sub>2</sub>7



Theo kết quả ?4 phương trình có hai
nghiệm


x1 = 4 14


2


 <sub> ; x</sub>


2 = 4 14


2




?7. Giải phương trình 2x2<sub> - 8x = -1</sub>


Chia cả hai vế cho 2, ta có: x2<sub> - 4x = - </sub>


2
1


Từ kết quả trên phương trình có hai
nghiệm


</div>
<span class='text_page_counter'>(12)</span><div class='page_container' data-page=12>

HS đọc SGK ví dụ 3.


Gọi HS trình bày lại bài giải.


GV: 2x2<sub> - 8x + 1 = 0 là một phương trình</sub>



bậc hai đủ. Khi giải phương trình ta đã
biến đổi để vế trái là bình phương của
một biểu thức chứa ẩn, vế phải là 1 hằng
số. Từ đó tiếp tục giải phương trình.


x1 = 4 14


2


 <sub> ; x</sub>


2 = 4 14


2




VD3: Giải phương trình.
2x2<sub> - 8x + 1 = 0</sub>


 2x2 - 8x = -1


 x2 - 4x = - <sub>2</sub>1


 x2 - 2. x . 2 + 22 = - <sub>2</sub>1 + 4


 ( x- 2) 2 = <sub>2</sub>7  x - 2 =  <sub>2</sub>7


 x - 2 = 


2
14


Vậy phương trình có hai nghiệm :
x1 = 4 14


2


 <sub> ; x</sub>


2 = 4 14


2



<b>Hoạt động 4 </b> <b>Hướng dẫn về nhà</b> (2’)


- Qua các VD trên, hãy nhận xét về số nghiệm của phương trình bậc 2.
- Làm bài tập 11, 12, 13 ,14 ( SGK).15, 16 (SBT)


- Tiết sau luyện tập


<i> Diễn Bích, ngày tháng năm 2010</i>
BGH kí duyệt


<i>Ngày soạn:</i>

<sub>31/01/2010</sub>

<i>Ngày dạy:</i>

<sub>1/02/2010</sub>



Tiết: 51 <b>LUYỆN TẬP</b>


I. <b>Mục tiêu</b>



- HS được củng cố lại khái niệm phương trình bậc hai một ẩn, xác định thành thạo
các hệ số a, b, c ; đặc biệt là a  0.


- Giải thành thạo các phương trình thuộc hai dạng đặc biệt khuyết b: ax2<sub> + c = 0 và </sub>


khuyết c: ax2<sub> + bx = 0.</sub>


- Biết và hiểu cách biến đổi một số phương trình có dạng tổng qt ax2<sub> + bx + c = 0 </sub>


( a  0 ) để được một phương trình có vế trái là bình phương, vế phải là hằng số.


II. <b>Chuẩn bị</b>


GV: Bảng phụ.


HS: Ơn các phương pháp giải phương trình


<b>III. Tiến trình dạy - học</b>
<b>Hoạt động 1</b> (8’)


HS1: Nêu định nghĩa phương trình bậc
hai một ẩn số và cho VD phương trình
bậc hai một ẩn? Hãy chỉ rõ hệ số a,b,c
của phương trình.


Giải phương trình sau 5x2<sub> - 20 = 0</sub>


<b>Kiểm tra:</b>



HS1:


ĐS: x =  2.


</div>
<span class='text_page_counter'>(13)</span><div class='page_container' data-page=13>

HS2: Giải phương trình 2x2<sub> + </sub> <sub>2</sub><sub>x = 0</sub>


HS2: ĐS : x1 = 0 ; x2 = -
2


2


Hoạt động 2 (35’)
GV nêu bài toán.


GV: Nhận xét dạng của phương trình
GV: Để giải phương trình khuyết c ta
thực hiện như thế nào? áp dụng kiến thức
nào?


Gọi 2 HS lên bảng giải.


GV nêu bài tốn.


GV: Nhận xét dạng của phương trình?
GV: Ta giải phương trình ở câu a như
thế nào?


HS trình bày cách giải câu b.


GV: Có cách giải khác khơng?


GV nêu bài tốn.


GV: Bài tốn u cầu làm gì?


GV: Muốn viết vế trái thành dạng bình
phương ta phải thêm vào số nào?


GV: Câu b ta giải như thế nào?


<b>Luyện tập:</b>


<b>Dạng 1 :</b> Giải phương trình
Bài 1. Giải các phương trình


a, 7x2<sub> - 5x = 0 </sub><sub></sub><sub> x( 7x - 5) = 0</sub>
 x = 0 hoặc 7x - 5 = 0
 x = 0 hoặc 7x = 5
 x = 0 hoặc x = <sub>7</sub>5


Vậy phương trình có hai nghiệm là
x1 = 0 và x2 = <sub>7</sub>


5


.


b, 3,4x2<sub> + 8,2 x = 0 </sub><sub></sub><sub> 34 x</sub>2<sub> + 82 x = 0 </sub>


 2x( 17x + 41) = 0



 2x = 0 hoặc 17x + 41 = 0
 x = 0 hoặc 17x = - 41.
 x = 0 hoặc x = -<sub>17</sub>41.


Vậy phương trình có 2 nghiệm là:
x1 = 0 ; x2 = -<sub>17</sub>


41


Bài 2. Giải các phương trình
a, - 3x2<sub> + 15 = 0</sub>


 - 3x2 = - 15  x 2 = 5
 x =  5


Vậy phương trình có hai nghiệm là :
x1 = 5 và x2 = - 5.


b, 1172,5x2<sub> + 42,18 = 0</sub>


vì 1172,5x2 <sub></sub><sub> 0 với mọi x </sub>


 1172,5x2 + 42,18 > 0 với mọi x.


 Vế trái không bằng vế phải với mọi x .


Vậy phương trình vơ nghiệm.


Bài 3. Giải các phương trình sau bằng
cách biến đổi chúng thành những phương


trình với vế trái là một bình phương cịn
vế phải là một hằng số.


a, x2<sub> + 8x = -2</sub>


 x2 + 8x + 16 = -2 + 16
 ( x + 4)2 = 14  x + 4 =  14


 x = - 4  14


Vậy phương trình có hai nghiệm
x1 = - 4 + 14 hoặc x2 = - 4 - 14 .


b, 2x2<sub> + 5x + 2 = 0</sub>


 2x2 + 5x = - 2  x2 + <sub>2</sub>5 x = -1


</div>
<span class='text_page_counter'>(14)</span><div class='page_container' data-page=14>

Bài 1 : Kết luận sai là :


a, Phương trình bậc hai một ẩn số


ax2<sub> + bx + c = 0 phải ln có điều kiện a </sub>
 0.


b, Phương trình bậc hai một ẩn khuyết c
khơng thể vơ nghiệm.


c, Phương trình bậc hai một ẩn khuyết cả
b và c ln có nghiệm.



d, Phương trình bậc hai khuyết b khơng
thể vơ nghiệm.


 x2 + 2 . x . <sub>4</sub>5 + <sub>16</sub>25 = -1 + <sub>16</sub>25


2


4
5











<i>x</i> =


16
9




5 3


4 4



5 3


4 4


<i>x</i>
<i>x</i>




 




  














2
2


1


<i>x</i>
<i>x</i>


Vậy phương trình có hai nghiệm
x1 = <sub>2</sub>


1


, x2 = -2


<b>Dạng 2 :</b> Bài tập trắc nghiệm.
Bài 1: Chọn d


Kết luận này sai vì phương trình bậc hai
khuyết b có thể vơ nghiệm.


VD: 2x2<sub> + 1 = 0 </sub>


<b>Hoạt động 3</b> <b>Hướng dẫn về nhà</b> (2’)
- Làm bài tập 16, 17, 18 ( SBT)


- Đọc trước bài “ Công thức nghiệm của phương trình bậc hai”


<i>Ngày soạn:</i>

<sub>2/02/2010</sub>

<i>Ngày dạy:</i>

<sub>3/02/2010</sub>



Tiết: 52 §4. <b>CƠNG THỨC NGHIỆM CỦA PHƯƠNG TRÌNH BẬC HAI</b>


I. <b>Mục tiêu</b>



- HS nhớ biệt thức  = b2 - 4ac và nhớ kĩ các điều kiện của  để phương trình bậc hai


một ẩn vơ nghiệm, có nghiệm kép, có hai nghiệm phân biệt.


- HS nhớ và vận dụng được cơng thức nghiệm tổng qt của phương trình bậc hai vào
giải phương trình ( chú ý khi a, b trái dấu, phương trình có hai nghiệm phân biệt).
II. <b>Chuẩn bị</b>


GV: Bảng phụ ghi ?1, Kết luận.
HS : MTBT.


<b>III. Tiến trình dạy - học</b>
<b>Hoạt động 1</b> (8’) <b>Kiểm tra:</b>


HS1: Giải phương trình sau bằng cách biến đổi chúng thành phương trình có vế trái
là một bình phương, còn vế phải là một hằng số.


3x2<sub> - 12x + 1 = 0</sub>


Giải.


3x2<sub> - 12x + 1 = 0 </sub>


 3x2 - 12 x = -1 ( chuyển 1 sang vế phải)


 x2 - 4x = -<sub>3</sub>1 ( chia hai vế cho 3)


</div>
<span class='text_page_counter'>(15)</span><div class='page_container' data-page=15>

 x2 - 2 . x . 2 + 4 = 4 - <sub>3</sub>1 ( tách 4x ở vế trái thành 2.x.2 và thêm vào hai



vế cùng một số để vế trái thành một bình phương)
 ( x - 2) 2 = 11<sub>3</sub>  x - 2 = 


3
11


 x - 2 = 
3
33 <sub> </sub>


 x = 2 +
3
33 <sub> =</sub>


3
33


6 <sub> hoặc x = 2 - </sub>


3
33 <sub> = </sub>


3
33
6 <sub> </sub>
Vậy phương trình có hai nghiệm x1 =


3
33
6 <sub>; x</sub>



2 =
3


33
6 <sub>.</sub>


GV : * ở bài trước, ta đã biết cách giải phương trình bậc hai một ẩn. Trong bài học
hơm nay, ta sẽ xét xem khi nào phương trình bậc hai có nghiệm và tìm cơng thức
nghiệm khi phương trình có nghiệm.


<b>Hoạt động 2</b> (20’)


GV : Cho phương trình ax2<sub> + bx + c = 0</sub>


( a  0 )


GV đưa các bước xây dựng công thức
nghiệm lên bảng phụ.


HS quan sát, so sánh cách làm với phần
hỏi bài cũ.


GV giới thiệu biệt thức  = b2 - 4ac.


* GV: Vế trái của phương trình (2) là số
khơng âm, vế phải có mẫu dương (4a2<sub> > 0</sub>


vì a  0 ), cịn tử thức là  có thể dương,



âm, bằng 0. Vậy nghiệm của phương
trình phụ thuộc vào , bằng hoạt động


nhóm hãy chỉ ra sự phụ thuộc đó.
HS thảo luận nhóm ?1,


GV: Gọi đại diện một nhóm lên bảng làm
?1.


Nhóm khác nhận xét.


<b>1. Cơng thức nghiệm:</b>


Cho phương trình ax2<sub> + bx + c = 0</sub>


( a  0 )


Biến đổi phương trình tổng quát


ax2<sub> + bx + c = 0 ( a </sub><sub></sub><sub> 0 ) ( 1)</sub>
 ax2 + bx = - c ( chuyển hạng tử tự do


sang vế phải)


 x2 + <i><sub>a</sub>b</i> x = - <i><sub>a</sub>c</i> ( vì a  0, chia hai vế


cho hệ số a )


 x2 + 2. x . <sub>2</sub><i>b<sub>a</sub></i> +



2


2 




<i>a</i>
<i>b</i>


= <sub>2</sub> 2





<i>a</i>
<i>b</i>


- <i><sub>a</sub>c</i>
( Tách hạng tử <i>b<sub>a</sub></i> x thành 2. x. <sub>2</sub><i>b<sub>a</sub></i> và
thêm vào hai vế cùng một biểu thức để vế
trái thành bình phương một biểu thức)




2


2 






<i>a</i>
<i>b</i>


<i>x</i> = <sub>2</sub>


2


4
4


<i>a</i>
<i>ac</i>


<i>b</i>  <sub> ( 2)</sub>


người ta kí hiệu  = b2 - 4ac


?1. Hãy điền những biểu thức thích hợp
vào các chỗ trống (...) dưới đây.


a, Nếu  > 0 thì phương trình (2) suy ra


x +


<i>a</i>
<i>b</i>



2 =  2<i>a</i>




Do đó, phương trình ( 1) có hai nghiệm :
x1 =


<i>a</i>
<i>b</i>


2





 <sub>; x</sub>


2=
<i>a</i>
<i>b</i>
2




b, Nếu  = 0 thì phương trình (2)


</div>
<span class='text_page_counter'>(16)</span><div class='page_container' data-page=16>

HS làm ?2. (câu c)



Nếu  < 0 thì vế phải của phương trình


( 2) là một số âm cịn vế trái là số khơng
âm nên phương trình (2) vơ nghiệm, do
đó phương trình (1) vơ nghiệm.


GV nhận xét bài làm của các nhóm.
GV nêu kết luận chung .


HS đọc kết luận.


GV: Đối với phương trình bậc hai, em có
nhận xét gì về số nghiệm của phương
trình?


suy ra x + <sub>2</sub><i>b<sub>a</sub></i> = 0.


Do đó, phương trình ( 1) có nghiệm kép
x1= x2 = - <i><sub>a</sub></i>


<i>b</i>


2 .


c, Nếu  < 0 thì phương trình (2) vơ


nghiệm.


Do đó phương trình (1) vơ nghiệm
Kết luận chung:



<b>Hoạt động 3</b> (15’)


GV nêu ví dụ SGK lên bảng phụ


* Để giải phương trình bậc hai bằng công
thức nghiệm , ta thực hiện qua các bước
nào?


HS : Ta thực hiện các bước sau:
+ Xác định hệ số a, b, c.


+ Tính  = b2 - 4ac


+ Tính nghiệm theo cơng thức nếu 


0


Kết luận phương trình vơ nghiệm nếu


 < 0.


* GV: Có thể giải mọi phương trình bậc
hai một ẩn bằng cơng thức nghiệm .
HS hoạt động nhóm làm ?3.


Gọi 3 HS lên bảng


<b>2. áp dụng:</b>



Ví dụ . Giải phương trình:
3x2<sub> + 5x - 1 = 0</sub>


?3. áp dụng cơng thức nghiệm để giải các
phương trình :


a, 5x2<sub> - x + 2 = 0</sub>


a = 5; b = -1; c = 2
 = b2 - 4ac


= (- 1) 2<sub> - 4. 5. 2 </sub>


= 1 - 40 = -39 < 0  PTVN.


b, 4x2<sub> - 4x + 1 = 0</sub>


129


Đối với phương trình ax2<sub> + bx + c = 0 ( a</sub>
 0 ) và biệt thức  = b2 - 4ac:


 Nếu  > 0 thì phương trình có hai


nghiệm phân biệt:
x1 =


<i>a</i>
<i>b</i>



2





 <sub>; x</sub>


2=
<i>a</i>
<i>b</i>


2






 Nếu  = 0 thì phương trình có nghiệm


kép


x1= x2 = - <i><sub>a</sub></i>
<i>b</i>


2 .


</div>
<span class='text_page_counter'>(17)</span><div class='page_container' data-page=17>

HS khác nhận xét


GV: Đối với phương trình b em nào có
cách giải khác ?



HS: 4x2<sub> - 4x + 1 = 0</sub>
 ( 2x - 1)2 = 0
 2x - 1 = 0
 x = - <sub>2</sub>1 .


* GV: Nếu chỉ là u cầu giải phương
trình ( khơng có u cầu “ áp dụng cơng
thức nghiệm” thì ta có thể chọn cách
nhanh hơn, ví dụ câu b .


GV: Em có nhận xét gì về hệ số a và c ở
phương trình câu c?


HS : a và c trái dấu


GV: Vì sao phương trình có a và c trái
dấu ln có hai nghiệm phân biệt ?
HS: Xét  = b2 - 4ac, nếu a và c trái dấu


thì tích ac < 0  - 4ac > 0


 = b2 - 4ac > 0  phương trình có hai


nghiệm phân biệt.


* GV : Nếu phương trình có hệ số a < 0
( như câu c) ta có thể nhân hai vế của
phương trình với ( -1) để a > 0 thì việc
giải phương trình thuận lợi hơn .



* GV khẳng định : Có thể giải mọi
phương trình bậc hai một ẩn bằng công
thức nghiệm nhưng với phương trình bậc
hai khuyết ta nên giải theo cách đưa về
phương trình tích hoặc biến đổi vế trái


a = 4; b = - 4; c = 1
 = b2 - 4ac


= (- 4) 2<sub> - 4. 4. 1 </sub>


= 16 - 16 = 0


Phương trình có nghiệm kép
x1 = x2 = -


<i>a</i>
<i>b</i>


2 = 2


1
4
.
2


4





c, - 3x2<sub> + x + 5 = 0</sub>


a = -3; b = 1; c = 5
 = b2 - 4ac


= 1 2<sub> - 4. (-3) . 5 </sub>


= 1 + 60 = 61 > 0


Phương trình có hai nghiệm phân biệt
x1=


6
61
1





 <sub> = </sub>


6
61
1 <sub> ; </sub>
x2=


6
61
1


6


61


1 







 <sub>.</sub>


Chú ý: ( SGK)


</div>
<span class='text_page_counter'>(18)</span><div class='page_container' data-page=18>

thành bình phương một biểu thức.


<b>Hoạt động 4 </b> <b>Hướng dẫn về nhà</b> (2’)
- Học thuộc “ Kết luận chung” trang 44 . SGK.
- Làm bài tập số 15, 16 SGK tr45.


- Đọc phần “ Có thể em chưa biết” tr46 . SGK.


<i> Diễn Bích, ngày tháng năm 2010</i>
BGH kí duyệt


<i>Ngày soạn:</i>

<sub>07/02/2010</sub>

<i>Ngày dạy:</i>

<sub>08/02/2010</sub>



Tiết: 53 <b>LUYỆN TẬP</b>



I. <b>Mục tiêu</b>


- HS nhớ kỹ các điều kiện của  để phương trình bậc 2 một ẩn vơ nghiệm, có nghiệm


kép, có hai nghiệm phân biệt.


- HS vận dụng công thức nghiệm tổng quát vào giải phương trình bậc hai một cách
thành thạo.


- HS biết linh hoạt với các trường hợp phương trình bậc hai đặc biệt không cần dùng
đến công thức nghiệm tổng quát.


II. <b>Chuẩn bị</b>


GV : Bảng phụ, MTBT.
HS: MTBT.


<b>III. Tiến trình dạy - học</b>


<b>Hoạt động 1</b> (10’) <b>Kiểm tra:</b>


HS: Điền vào chỗ có dấu .... để được kết luận đúng:


Đối với phương trình ax2<sub> + bx + c = 0 ( a </sub><sub></sub><sub> 0 ) và biệt thức </sub><sub></sub><sub> = b</sub>2<sub> - 4ac:</sub>


* Nếu  .... thì phương trình có 2 nghiệm phân biệt :


x1 = ...; x2 = ...


* Nếu  ... thì phương trình có nghiệm kép x1 = x2 = ....



* Nếu  .... .. thì phương trình vơ nghiệm .


Làm bài 15(b,d).Khơng giải phương trình, hãy xác định hệ số a, b,c , tính  và tìm số


nghiệm của mỗi phươnng trình.


b,  = 0  phương trình có nghiệm kép.


d,  = 15,72 > 0  phương trình có 2 nghiệm phân biệt.


</div>
<span class='text_page_counter'>(19)</span><div class='page_container' data-page=19>

<b>Hoạt động 2</b> (33’)


Dạng 1 . Giải phương trình bằng cơng
thức nghiệm:


Gọi 3 HS lên bảng


GV: Với câu b có thể giải cách khác
nhanh hơn khơng?


Dạng 2 . Giải phương trình bằng đồ
thị:


GV nêu bài 2


GV: Gọi 2 HS lên bảng lập bảng toạ
độ điểm rồi vẽ đồ thị


<b>Luyện tập:</b>



Bài 1 Giải các phương trình
a, 2x2<sub> - ( 1 - 2</sub> <sub>2</sub> <sub>) x - </sub> <sub>2</sub><sub> = 0</sub>


a = 2; b = -( 1 - 2 2) ; c = - 2
 = b2 - 4ac


= ( 1 - 2 2)2 - 4 . 2 . ( - 2 )


= 1 - 4 2 + 8 + 8 2


= 1 + 4 2 + 8 = ( 1+ 2 2)2 > 0
  = 1 + 2 2


Vậy phương trình có hai nghiệm phân biệt


x1 = 1 2 2 1 2 2 1


4 2


  




x2 = 1 2 2 1 2 2 2


4


  






b, 4x2<sub> + 4x + 1 = 0</sub>


 = 16 - 16 = 0 phương trình có nghiệm kép


x1=x2=-


2
1
8
4


2<i>a</i>  


<i>b</i>


C2: 4x2 + 4x + 1 = 0


 ( 2x + 1) 2 = 0


 2x = - 1


 x = - <sub>2</sub>1


c, - 3x2<sub> + 2x + 8 = 0 </sub>


 3x2- 2x – 8 = 0


 = b2 - 4ac = ( -2)2 - 4. 3 . ( -8)



= 4 + 96 = 100 > 0


  = 10


Phương trình có hai nghiệm phân biệt
x1 = <sub>6</sub>


10
2


= 2 ; x2= <sub>6</sub>


10
2


= - <sub>3</sub>4
Bài 2 Giải phương trình bằng đồ thị
Cho phương trình 2x2<sub> + x - 3 = 0 (1)</sub>


a, Vẽ đồ thị y = 2x2<sub> ; y = - x + 3</sub>


trong cùng một mặt phẳng toạ độ
* y = 2x2


x -2,5 -2 -1 0 1 2 3


y = 2x2 <sub>12,5</sub> <sub>8</sub> <sub>1 0 2 8 12,5</sub>


* y = - x + 3



x 0 3


y = -x + 3 3 0


Hai đồ thị này cắt nhau tại A ( -1,5; 4,5 )
và B ( 1; 2)


132


-1,5 O 1 3 x
y


4,5




3
2
A


</div>
<span class='text_page_counter'>(20)</span><div class='page_container' data-page=20>

GV: Hãy tìm toạ độ giao điểm của hai
đồ thị?


GV: Hãy giải thích vì sao x1 = - 1,5 là


nghiệm của phương trình (1)


GV: Tương tự giải thích vì sao x2 = 1



là nghiệm của phương trình (1)?


GV: Hãy giải phương trình bằng cơng
thức nghiệm? So sánh với kết quả của
câu b?


Dạng 3. Tìm điều kiện của tham số
Bài 3. Đối với mỗi phương trình sau,
hãy tìm các giá trị của m để phương
trình có nghiệm; tính nghiệm của
phương trình theo m:


a, mx2<sub> + ( 2m - 1) + m + 2 = 0 </sub>


GV: Phương trình bậc hai có nghiệm
khi nào?


GV: Hãy tính ?


b, x1 = -1,5 ; x2 = 1


x1= 1,5 là nghiệm của phương trình (1)


vì 2. ( -1,5 )2<sub> + ( - 1,5 ) - 3 </sub>


= 2. 2,25 - 1,5 - 3
= 4,5 - 4,5 = 0


Tương tự ta có x = 1 là nghiệm của phương
trình (1).



c, 2x2<sub> + x - 3 = 0</sub>


 = 1 + 4. 2 . (-3) = 25 > 0


Vậy phương trình (1) có hai nghiệm phân
biệt:


x1 = <sub>4</sub>


5
1


= 1; x2 = <sub>4</sub>


5
1


= -1,5
Kết qủa trùng với kết quả câu b.


Bài 3. Giải


a, mx2<sub> + ( 2m - 1) + m + 2 = 0</sub>


ĐK : m  0


 = ( 2m - 1)2 - 4m ( m + 2)



= 4m2<sub> - 4m + 1 - 4m</sub>2<sub> - 8m = -12 m + 1</sub>


Phương trình có nghiệm   0


hay - 12m + 1  0


 - 12 m  - 1  m  <sub>12</sub>1


với m  <sub>12</sub>1 và m  0 thì phương trình (1)


có nghiệm .


Nghiệm của phương trình là:
x1 =


<i>m</i>


<i>m</i>
<i>m</i>


2


12
1
2


1   <sub> ; x</sub>


2 =



<i>m</i>


<i>m</i>
<i>m</i>


2


12
1
2


1  
b, 3x2<sub> + ( m + 1) x - 4 = 0 ( 2)</sub>


 =( m + 1) 2 + 4 . 3. 4 = ( m + 1)2 + 48 > 0


vì  > 0 với mọi giá trị của m do đó phương


trình (2) có nghiệm với mọi giá trị của m.


</div>
<span class='text_page_counter'>(21)</span><div class='page_container' data-page=21>

b, 3x2<sub> + ( m + 1) x - 4 = 0 ( 2)</sub>


GV: Phương trình có nghiệm khi nào?
GV: Vậy nghiệm của phương trình?
GV: Tương tự HS giải câu b.


Nghiệm của phương trình là :
x1 =



6


48
)
1
(


1 2








 <i>m</i> <i>m</i>


x2 =


6


48
)
1
(


1 2








 <i>m</i> <i>m</i>


<b>Hoạt động 3 </b> <b>Hướng dẫn về nhà</b> (2’)
- Làm bài tập 21,23,24 ( SBT)


- Đọc bài đọc thêm: Giải phương trình bậc hai bằng MTBT.


<i>Ngày soạn:</i>

<sub>08/02/2010</sub>

<i>Ngày dạy:</i>

<sub> / /2010</sub>



Tiết:54 § 5<i><b>. </b></i><b>CÔNG THỨC NGHIỆM THU GỌN</b>


I. <b>Mục tiêu</b>


- HS thấy được lợi ích của cơng thức nghiệm thu gọn.


- HS biết tìm b’ và biết tính ’, x1, x2 theo công thức nghiệm thu gọn.


- HS nhớ và vận dụng tốt công thức nghiệm thu gọn.
II. <b>Chuẩn bị</b>


GV: Bảng phụ viết sẵn hai bảng công thức nghiệm của phương trình bậc hai.
HS: MTBT.


<b>III. Tiến trình dạy - học</b>


Hoạt động 1 (8’)



HS1: Giải phương trình bằng công thức
nghiệm 3x2<sub> + 8x + 4 = 0</sub>


<b>Kiểm tra:</b>


HS1:


a = 3; b = 8; c = 4


 = 82 – 4.3.4 = 64 – 48 = 16


4


 


Phương trình có hai nghiệm phân biệt:


1


8 4 2


2 3 3


x


.


  


  ; <sub>2</sub> 8 4 2



2 3


x


.
 


 


<b>Hoạt động 2</b> (15’)


GV : Đối với phương trình


ax2<sub> + bx + c = 0 ( a </sub><sub></sub><sub> 0), trong nhiều </sub>


trường hợp nếu đặt b = 2b’ rồi áp dụng
công thức nghiệm thu gọn thì việc giải
phương trình sẽ đơn giản hơn.


Trước hết ta xây dựng công thức nghiệm
thu gọn.


Nếu đặt b = 2b’


GV: Hãy tính biệt số  theo b’?


1. <b>Cơng thức nghiệm thu gọn:</b>


Cho phương trình ax2<sub> + bx + c = 0 (a </sub>


 0)


Đặt b = 2b’


thì  = b2 - 4ac = (2b’)2 - 4ac


</div>
<span class='text_page_counter'>(22)</span><div class='page_container' data-page=22>

GV kí hiệu ’ = b’2 - ac thì  = ?


GV: Căn cứ vào cơng thức nghiệm đã
học


b = 2b’ và  = 4’ hãy tìm nghiệm của


phươnng trình bậc hai ( nếu có) với
trường hợp ’ > 0; ’ = 0; ’ < 0.


HS thảo luận nhóm làm bài tập .


Điền vào các chỗ trống (...) để được kết
quả đúng.


* Nếu ’ > 0 thì  > ...
  = ... 


phương trình có ...
x1 =


<i>a</i>
<i>b</i>
2




 <sub>= </sub>
<i>a</i>
<i>b</i>
2
2


2  


 <sub>= </sub>


<i>a</i>


....
....


x2 = <sub>...</sub>


....
....


= ....<sub>...</sub> .... = ....<sub>...</sub> ....
* Nếu ’ = 0 thì  .... phương trình có ....


x1 = x2 =
<i>a</i>
<i>b</i>
2


=
<i>a</i>
2
...
=
...
...
.


* Nếu ’ < 0 thì  ... phương trình ....


GV đưa lên bảng phụ hai bảng cơng thức
nghiệm


GV hướng dẫn HS so sánh hai công thức
tương ứng để ghi nhớ và nhấn mạnh cho
HS thấy  và ’ cùng dấu vì  = 4’


nên số nghiệm của phương trình khơng


= 4b’2<sub> - 4ac</sub>


= 4 ( b’2<sub> - ac)</sub>


Kí hiệu


ta có  = 4’


Đáp án:



 Nếu ’ > 0 thì  > 0
  = 2 


phương trình có hai nghiệm phân biệt:
x1 =


<i>a</i>
<i>b</i>
2


 <sub>= </sub>
<i>a</i>
<i>b</i>
2
2


2  


 <sub>= </sub>


<i>a</i>
<i>b</i> 


x2 =
<i>a</i>
<i>b</i>
2



 <sub>= </sub>
<i>a</i>
<i>b</i>
2
2


2  


 <sub>= </sub>


<i>a</i>
<i>b</i> 


 <sub> </sub>


 Nếu ’ = 0 thì = 0 phương trình có


nghiệm kép
x1= x2 =


2


b
a


= <sub>2</sub>2<i><sub>a</sub>b</i>= <i><sub>a</sub>b</i>.


Nếu ’ < 0 thì  < 0 phương trình vơ



nghiệm.


135


’ = b’2 - ac


<b>Công thức nghiệm của phương trình</b>
<b>bậc hai</b>


<b>Cơng thức nghiệm thu gọn của</b>
<b>phươngtrình bậc hai.</b>


Đối với phương trình :
ax2<sub> + bx + c = 0 ( a </sub><sub></sub><sub> 0)</sub>


Đối với phương trình :
ax2<sub> + bx + c = 0 ( a </sub><sub></sub><sub> 0)</sub>


b = 2b’


 = b2 - 4ac ’ = b’2 - ac


 Nếu  > 0 thì phương trình có hai


nghiệm phân biệt:
x1 =


<i>a</i>
<i>b</i>



2





 <sub>; x</sub>


2=
<i>a</i>
<i>b</i>
2




 Nếu ’ > 0 thì phương trình có hai


nghiệm phân biệt:
x1 =


<i>a</i>
<i>b</i> 
 ' <sub>; x</sub>


2=
<i>a</i>
<i>b</i> 
 '
 Nếu  = 0 thì phương trình có nghiệm



kép


x1= x2 = - <i><sub>a</sub></i>
<i>b</i>


2 .


 Nếu ’ = 0 thì phương trình có


nghiệm kép
x1= x2 = -


<i>a</i>
<i>b</i>


.


</div>
<span class='text_page_counter'>(23)</span><div class='page_container' data-page=23>

thay đổi dù xét  hay ’.
<b>Hoạt động 3</b> (10’)
HS làm ?2


Gọi HS lên bảng giải.


HS làm ?3


Gọi 2 HS lên bảng.


GV: Khi nào ta nên dùng công thức
nghiệm thu gọn?



HS: Ta nên dùng cơng thức nghiệm thu
gọn khi phương trình bậc hai có b là số
chẵn hoặc là bội chẵn của một căn, một
biểu thức.


<b>2. áp dụng:</b>


?2. Giải phương trình 5x2<sub> + 4x - 1 = 0 </sub>


bằng cách điền vào chỗ trống
a = 5; b’ <sub> = 2; c = -1</sub>


’ = b’2 - ac = 4 + 5 = 9 = 3


Nghiệm của phương trình :
x1 =


5
1
5
3
2




x2 = 1


5


3
2





?3. Xác định a, b’, c rồi dùng công thức
nghiệm thu gọn giải các phương trình
a, 3x2<sub> + 8x + 4 = 0</sub>


a = 3; b’ = 4; c = 4


’ = b’2 - ac = 42 - 3.4 = 16 - 12 = 4
= 2


Nghiệm của phương trình :


x1 = <sub>3</sub>


2
3
2
4 




; x2 = <sub>3</sub> 2



2
4





b, 7x2<sub> - 6 </sub> <sub>2</sub><sub>x + 2 = 0 ; </sub>


a = 7; b’ = -3 2; c = 2


’ = b’2- ac = (-3 2)2 - 7.2 = 18 - 14 = 4


= 2


Nghiệm của phương trình :
x1 =


7
2
2


3  <sub>; x</sub>


2 =
7


2
2
3 



<b>Hoạt động 4</b> (10’)
HS làm bài 17(c,d)


GV gọi 2 HS lên bảng cùng làm


<b>Luyện tập:</b>
<b>Bài 17-SGK</b>


c) <sub>5</sub><sub>x</sub>2<sub></sub> <sub>6</sub><sub>x</sub><sub> </sub><sub>1 0</sub>


a = 5; b’ = -3 ; c = 1


2


3 5 1 9 5 4


' ( ) .


      


2


'


 


Phương trình có hai nghiệm:


1



3 2
1
5


x    ; <sub>2</sub> 3 2 1


5 5


x   


d) <sub></sub><sub>3</sub><sub>x</sub>2<sub></sub><sub>4 6</sub><sub>x</sub><sub> </sub><sub>4 0</sub>


a = -3 ; b '2 6; c = 4


2


2 6 3 4 24 12 36


' ( ) ( ).


      


6


'


 


Phương trình có hai nghiệm:



</div>
<span class='text_page_counter'>(24)</span><div class='page_container' data-page=24>

1


2 6 6
3


x  


 ; 2


2 6 6
3


x  



<b>Hoạt động 5 </b> <b>Hướng dẫn về nhà</b> (2’)


- Học nắm chắc hai công thức nghiệm, biết áp dụng công thức nghiệm để giải phương
trình bậc hai.


- BTVN : 17(a,b), 18, 19, 20( SGK)
27, 28 (SBT)


<i>Ngày soạn:</i>

<sub>22/02/2010</sub>

<i>Ngày dạy:</i>

<sub>24/02/2010</sub>



Tiết: 55 <b>LUYỆN TẬP</b>


I. <b>Mục tiêu</b>



- HS thấy được lợi ích của cơng thức nghiệm thu gọn và thuộc kỹ công thức nghiệm
thu gọn.


- HS vận dụng thành thạo công thức này để giải để giải phương trình bậc hai.
- Kiểm tra 15’ về phương trình bậc hai.


II. <b>Chuẩn bị</b>


GV: Bài kiểm tra photo


HS: Ơn cơng thức nghiệm và cơng thức nghiệm thu gọn.


<b>III. Tiến trình dạy - học</b>
<b>Hoạt động 1</b> (28’)


HS làm bài 20-SGK (HS hoạt động
nhóm)


GV: Em có nhận xét gì về phương trình
ở câu a, b, c? (khuyết b, khuyết c))
GV: Giải phương trình này như thế nào?
GV: Gọi HS lên bảng giải


GV : Với phương trình bậc hai khuyết,
nhìn chung khơng nên giải bằng cơng
thức nghiệm mà nên đưa về phương trình
tích hoặc dùng cách giải riêng.


HS làm bài tập 21 ( SGK)
Gọi 2 HS lên bảng giải.



<b>Luyện tập:</b>


<b>Dạng 1: Giải phương trình .</b>


Bài 20 ( SGK)
a, 25x2<sub> - 16 = 0</sub>


 25x2 = 16  x2 = 16<sub>25</sub>  x = <sub>5</sub>4


b, 2x2<sub> + 3 = 0</sub>


vì 2x2


 0 với mọi x  2x2 + 3 > 0 với


mọi x


Vậy phương trình vơ nghiệm.
c, 4,2x2<sub> + 5,46x = 0</sub>


x ( 4,2x + 5,46 ) = 0  x = 0


hoặc 4,2x + 5,46 = 0  x =-5<sub>4</sub>,<sub>,</sub>46<sub>2</sub>


 x = -1,3


Vậy phương trình có hai nghiệm : x1= 0;


x2 = -1,3.



Bài 21 ( SGK) Giải vài phương trình của
AnKhô-va-ri-zmi


a, x2<sub> = 12x + 288</sub>


x2<sub> - 12x - 288 = 0</sub>


</div>
<span class='text_page_counter'>(25)</span><div class='page_container' data-page=25>

HS làm bài 22-SGK


GV: Khơng giải phương trình, muốn xét
số nghiệm ta dựa vào cơ sở nào?


* GV nhấn mạnh lại nhận xét : Khi a,c
trái dấu  ac < 0 . Phương trình ln có


hai nghiệm phân biệt.
HS làm bài 24-SGK


GV: Hãy tính ’?


GV: Phương trình có hai nghiệm phân
biệt khi nào?


GV: Phương trình có nghiệm kép khi
nào?


GV: Phương trình vơ nghiệm khi nào?


a= 1, b’ = - 6, c = -288



’ = 36 + 288 = 324 > 0 = 18


Phương trình có hai nghiệm phân biệt:
x1 = 6 + 18 x1 = 24


x2 = 6 - 18  x2 = -12


b, <sub>12</sub>1 x2<sub> + </sub>


12
7


x = 19
 x2 + 7x - 288 = 0


 = 72 - 4 (-288) = 961   = 31
Phương trình có hai nghiệm phân biệt
x1 = 7 31 12


2


 


 ; x<sub>2</sub> = 7 31 19


2


 




<b>Dạng 2: Nhận xét số nghiệm.</b>


Bài 22 ( SGK)


Khơng giải phương trình, xét số nghiệm
của nó.


a, 15x2<sub> + 4x - 2005 = 0</sub>


có a = 15 > 0; c = -2005 < 0  ac < 0
 Phương trình có hai nghiệm phân biệt.


b, - 19<sub>5</sub> x2<sub>- </sub> <sub>7</sub><sub>x + 1890 = 0</sub>


Tương tự trên có a và c trái dấu  phương


trình có hai nghiệm phân biệt.


<b>Dạng 3: Tìm điều kiện của tham số.</b>


Bài 24 ( SGK)


Cho phương trình x2<sub> - 2( m - 1)x + m</sub>2<sub> = 0</sub>


a = 1, b’ = - (m -1) , c = m2


a, Tính ’ = (m -1)2 - m2


= m2<sub> - 2m + 1 - m</sub>2<sub> = 1 - 2m</sub>



b, Phương trình có hai nghiệm phân biệt


 ’ > 0  1 - 2m > 0  m < 1<sub>2</sub>


Phương trình có nghiệm kép  ’ = 0
 1 - 2m = 0  - 2m = -1  m = 1<sub>2</sub>


Phương trình vơ nghiệm  ’ < 0
 1 - 2m < 0  - 2m < - 1  m > 1<sub>2</sub>
<b>Hoạt động 2</b> (15’) <b>Kiểm tra 15’</b>


<b>A. Đề bài:</b>


Bài 1: Giải các phương trình sau:


a) x2<sub> + 4x – 5 = 0 b) x</sub>2<sub> - 8</sub> <sub>2</sub><sub>x - 4 = 0</sub>


Bài 2: Tìm m để phương trình có nghiệm kép:
x2<sub> – mx + 1 = 0</sub>


<b>B. Đáp án – Biểu điêm:</b>


<b>Bài 1:</b> Mỗi bài đúng cho 3 điểm.


</div>
<span class='text_page_counter'>(26)</span><div class='page_container' data-page=26>

a) x1 = 1; x2 = - 5 b) x1 4 2 6 ; x2 4 2 6


<b>Bài 2:</b> (4 điểm) m = ± 2


<b>Hoạt động 3 </b> <b>Hướng dẫn về nhà</b> (2’)



- Học thuộc công thức nghiệm, công thức nghiệm thu gọn, nhận xét sự khác nhau.
- BTVN : 29, 30, 32, 33 9 (SBT)


</div>
<span class='text_page_counter'>(27)</span><div class='page_container' data-page=27>

<i>Ngày soạn:</i>

<sub>28/02/2010</sub>

<i>Ngày dạy:</i>

<sub>01/03/2010</sub>



Tiết: 56 <b>§6. HỆ THỨC VI-ÉT VÀ ỨNG DỤNG</b>


I. <b>Mục tiêu</b>


+ HS nắm vững hệ thức Vi- ét.


+ HS vận dụng được những ứng dụng của hệ thức Vi-ét như :


- Nhẩm nghiệm của phương trình bậc hai trong các trường hợp a + b + c = 0;


a - b + c = 0 hoặc trường hợp tổng và tích của hai nghiệm là những số nguyên với giá
trị tuyệt đối không q lớn.


- Tìm được hai số biết tổng và tích của chúng.
II. <b>Chuẩn bị</b>


GV: Bảng phụ ghi các bài tập, MTBT.


HS: Ơn tập nghiệm tổng qt của phương trình bậc hai, MTBT.


<b>III. Tiến trình dạy - học</b>
<b>Hoạt động 1</b> (8’)


HS: Phương trình bậc hai ax2<sub> + bx + c = </sub>



0 ( a  0) có nghiệm khi nào?


Viết các nghiệm đó dưới dạng tổng quát.
Gọi 2 HS lên bảng ( cả lớp làm vào vở)
Hãy tính x1 + x2 ; x1 . x2


<b>Kiểm tra:</b>


HS:


Nếu phương trình bậc hai ax2<sub> + bx + c = 0</sub>


( a  0 ) có nghiệm thì ta có  ≥ 0


x1 =
<i>a</i>
<i>b</i>


2





 <sub>; x</sub>


2=
<i>a</i>
<i>b</i>
2






x1 + x2 =
<i>a</i>
<i>b</i>
2


 <sub> + </sub>
<i>a</i>
<i>b</i>
2


 <sub>= </sub>
<i>a</i>
<i>b</i>
2
2

=
<i>a</i>
<i>b</i>


x1 . x2 =
<i>a</i>
<i>b</i>


2


 <sub>.</sub>
<i>a</i>
<i>b</i>
2


 <sub> =</sub>
<i>a</i>
<i>c</i>


GV: Từ bài tốn trên ta thấy phương trình ax2<sub> + bx + c = 0 (a </sub>


 0) có nghiệm x1, x2


thì: x1 + x2 = <i><sub>a</sub></i>
<i>b</i>




<b>; </b>x1 . x2 = <i><sub>a</sub></i>
<i>c</i>


.Các hệ thức này thể hiện mối liên hệ giữa các nghiệm
với các hệ số của phương trình. Đây gọi là hệ thức Vi-ét. GV giới thiệu bài mới.


<b>Hoạt động 2</b> (15’)
HS đọc định lí.



GV nhấn mạnh : Hệ thức Vi - ét thể hiện
mối liên hệ giữa các nghiệm và các hệ số
của phương trình.


GV giới thiệu Nhà tốn học Pháp
Phzănăxoa Vi - ét ( 1540 - 1603).


GV: Điều kiện để có hệ thức Vi-ét là gì?
GV nhấn mạnh cho HS “ Phương trình có
nghiệm thì…”


<i><b>GV đưa bài tốn lên bảng phụ: </b></i>


Biết rằng các phương trình sau có
nghiệm, khơng giải phương trình, hãy
tính tổng và tích các nghiệm của chúng.
a, 2x2<sub> - 9x + 2 = 0</sub>


<b>1. Hệ thức Vi – ét:</b>


Nếu x1; x2 là hai nghiệm của phương


trình ax2<sub> + bx + c = 0 ( a </sub>


 0 ) thì


</div>
<span class='text_page_counter'>(28)</span><div class='page_container' data-page=28>

b, - 3x2<sub> + 6x - 1 = 0</sub>


GV áp dụng: Nhờ định lí Vi- ét, nếu đã


biết một nghiệm của phương trình bậc
hai, ta có thể suy ra nghiệm kia.


HS thảo luận nhóm ?2, ?3.
Mỗi nhóm làm một ?


Gọi đại diện 2 nhóm lên bảng trình bày.


GV: Qua ?2, ?3 em rút ra điều gì về
nghiệm của phương trình ax2<sub> + bx + c = 0</sub>


( a  0) khi có a + b + c = 0 ?


Tương tự khi có a - b + c = 0?


HS làm ?4.


Gọi 2 HS lên bảng.


a, 2x2<sub> - 9x + 2 = 0</sub>


x1 + x2 = - <i><sub>a</sub></i>
<i>b</i>


= <sub>2</sub>9 ; x1 . x2 = <i><sub>a</sub></i>
<i>c</i>


= <sub>2</sub>2 =
1



b, - 3x2<sub> + 6x - 1 = 0</sub>


x1 + x2 = - <i><sub>a</sub></i>
<i>b</i>


= <sub>3</sub>6





=2; x1 . x2 = <i><sub>a</sub></i>
<i>c</i>


= <sub>3</sub>1





=


3
1


?2. Cho phương trình 2x2<sub> - 5x + 3 = 0</sub>


a) a = 2; b = -5; c = 3.


a + b + c = 2 + ( -5) + 3 = 0
b) Thay x1 = 1 vào phương trình ta có



2 . 12<sub> - 5. 1 + 3 = 0</sub>


 x1= 1 là một nghiệm của phương trình.


c) Theo hệ thức Vi- ét
x1 . x2 = <i><sub>a</sub></i>


<i>c</i>


, có x1 = 1  x2 = <i><sub>a</sub></i>
<i>c</i>


= <sub>2</sub>3
?3. Cho phương trình 3x2<sub> + 7x + 4 = 0</sub>


a) a = 3; b = 7; c = 4


có a - b + c = 3 - 7 + 4 = 0
b) Thay x1 = -1 vào phương trình


3. ( - 1)2<sub> + 7. ( -1) + 4 = 0</sub>


 x1=-1 là một nghiệm của phương trình.


c) Theo hệ thức Vi- ét
x1. x2=


<i>a</i>
<i>c</i>



, có x1 = -1  x2 = -
<i>a</i>
<i>c</i>


= -


3
4


Tổng quát: Phương trình ax2<sub> + bx + c = 0 </sub>


( a  0 )


Nếu có a + b + c = 0 thì phương trình có
một nghiệm là x1 =1,cịn nghiệm kia là x2 =


<i>a</i>
<i>c</i>


Nếu có a - b + c = 0 thì phương trình có
một nghiệm là x1 =-1,còn nghiệm kia là x2 =- <i><sub>a</sub></i>


<i>c</i>


?4. Tính nhẩm nghiệm của các phương trình :
a, - 5x2<sub> + 3x + 2 = 0</sub>


Có a + b + c = -5 + 3 + 2 = 0


 x1 = 1; x2 = <i><sub>a</sub></i>


<i>c</i>


= -<sub>5</sub>2
b, 2004x2<sub> + 2005x + 1 = 0</sub>


Có a - b + c =2004 - 2005 + 1= 0


</div>
<span class='text_page_counter'>(29)</span><div class='page_container' data-page=29>

 x1 = -1 ; x2 = - <i><sub>a</sub></i>
<i>c</i>


= <sub>2004</sub> 1


<b>Hoạt động 3</b> (10’)


GV : Hệ thức Vi-ét cho ta biết cách tính
tổng và tích hai nghiệm của phương trình
bậc hai. Ngược lại nếu biết tổng của hai
số nào đó bằng S và tích của chúng bằng
P thì hai số đó có thể là nghiệm của một
phương trình nào chăng?


GV nêu bài toán


GV: Hãy chọn ẩn và lập phương trình bài
tốn?


GV: Phương trình này có nghiệm khi
nào?


GV hướng dẫn HS làm VD1 (bảng phụ).


HS làm ?5.


GV: Hai số cần tìm là nghiệm của
phương trình nào?


Gọi HS trình bày bài gải..


<b>2. Tìm hai số biết tổng và tích của </b>
<b>chúng:</b>


Bài tốn: Tìm hai số có tổng bằng S và
tích bằng P.


Giải.


Gọi một số là x thì số kia là S - x
Theo giả thiết ta có phương trình
x( S – x) = P hay x2<sub>- Sx + P = 0</sub>


Nếu  = S2 – 4P  0 thì phương trình (1)


có nghiệm. Các nghiệm này chính là hai
số cần tìm.


?5.Tìm hai số biết tổng bằng 1, tích bằng 5.
Hai số cần tìm là hai nghiệm của phương
trình x2<sub>- x + 5 = 0</sub>


 = (-1)2 – 4. 1. 5 = -19 < 0. Phương trình



vơ nghiệm.


Vậy khơng có hai số nào có tổng bằng 1
và tích bằng 5.


<b>Hoạt động 4</b> (10’)


HS làm bài 25(a,c) – SGK


GV: Qua bài tập 25 chúng ta cần lưu ý
điều gì?


HS: Hệ thức Vi-ét chỉ được ứng dụng cho
các phương trình có nghiệm.


HS làm bài 28a – SGK


GV: Điều kiện để có hai số cần tìm là gì?


<b>Luyện tập:</b>


Bài 25 – SGK
a) x2<sub> – 17x + 1 = 0</sub>


 = (-17)2 – 4.2.1 = 289 – 8 = 281 > 0


1 2


17



x x


2


  ; x .x<sub>1</sub> <sub>2</sub> 1


2




c) 8x2<sub> – x + 1 = 0</sub>


 = (-1)2 – 4.8.1 = 1 – 32 = - 31 < 0


Phương trình vơ nghiệm.


Bài 28a-SGK. Hai số cần tìm là nghiệm
của phương trình: x2<sub> + 8x – 105 = 0 </sub>
’ = 42 + 105 = 16 + 105 = 121;  ' 11


x1 = - 4 + 11 = 7; x2 = - 4 – 11 = - 15


Vậy hai số cần tìm là - 15 và 7


<b>Hoạt động 5 </b> <b>Hướng dẫn về nhà</b> (2’)


- Học thuộc hệ thức Vi-ét và ứng dụng, cách tìm hai số biết tổng và tích.
- Làm bài tập 25(b,d), 26, 27(a,b,d) , 28 ( SGK).


</div>
<span class='text_page_counter'>(30)</span><div class='page_container' data-page=30>

<i>Ngày soạn:</i>

<sub>02/03/2010</sub>

<i>Ngày dạy:</i>

<sub>04/03/2010</sub>




Tiết: 57 <b>LUYỆN TẬP</b>


I. <b>Mục tiêu</b>


* Củng cố hệ thức Ví - ét.


* Rèn kỹ năng vận dụng hệ thức Vi - ét để:
- Tính tổng, tích các nghiệm của phương trình.


- Nhẩm nghiệm của phương trình trong các trường hợp có a + b + c = 0, a - b +c = 0
hoặc qua tổng, tích của hai nghiệm (nếu hai nghiệm là những số nguyên có giá trị
tuyệt đối khơng q lớn)


- Tìm hai số biết tổng và tích của nó.


- Lập phương trình biết hai nghiệm của nó.


- Phân tích đa thức thành nhân tử nhờ nghiệm của đa thức.
II. <b>Chuẩn bị</b>


GV: Các dạng bài tập liên quan hệ thức Vi-ét.
HS: Ôn tập hệ thức Vi-ét.


<b>III. Tiến trình dạy - học</b>
<b>Hoạt động 1</b> (8’)


HS1:


- Phát biểu hệ thức Vi - ét.



- Chữa bài tập 27 : Dùng hệ thức Vi - ét
để nhẩm các nghiệm của phương trình
a, x2<sub> - 7x + 12 = 0</sub>


HS2:


- Nêu cách tính nhẩm nghiệm trường hợp
a + b + c = 0 và a - b + c = 0.


- Chữa bài tập 26c, d.
a, 35x2<sub> - 37x + 2 = 0</sub>


c, x2<sub> - 49x - 50 = 0</sub>


<b>Kiểm tra:</b>


HS1:


- Bài tập 27 :
a, x2<sub> - 7x + 12 = 0</sub>


ĐS : x1 = 3; x2 = 4.


HS2:


ĐS : a, x1 = 1; x2 = <sub>35</sub>


2



;
c, x1 = - 1; x2 = 50.


<b>Hoạt động 2</b> (33’) <b>Luyện tập:</b>


</div>
<span class='text_page_counter'>(31)</span><div class='page_container' data-page=31>

HS làm bài tập 30 ( SGK)


GV: Phương trình có nghiệm khi nào?
GV: ính tổng và tích các nghiệm theo m?
Gọi 1 HS lên bảng làm câu b.


HS làm bài tập 31.


GV: Bài tốn u cầu làm gì?


GV: Muốn nhẩm nghiệm ta áp dụng kiến
thức nào?


GV lưu ý : nhận xét xem với mỗi trường
hợp bài áp dụng được trường hợp


a + b + c = 0 hay a - b + c = 0.


HS làm bài tập 32 ( SGK)


GV: Từ giả thiết ta có điều gì?


GV: u và v là nghiệm của phương trình
nào?



GV: Giải phương trình tìm nghiệm?


Bài 30 ( SGK) Tính giá trị của m để
phương trình có nghiệm, rồi tính tổng và
tích các nghiệm theo phương trình.


a, x2<sub> - 2x + m = 0 </sub>


Giải.


’ = (-1)2 - m = 1 - m


Phương trình có nghiệm


’  0  1 - m  0  m  1.


- Theo hệ thức Vi- ét, ta có:
x1 + x2 = - <i><sub>a</sub></i>


<i>b</i>


= 2 ; x1 . x2 = <i><sub>a</sub></i>
<i>c</i>


= m
b, x2<sub> + 2(m - 1)x +m</sub>2<sub>= 0 </sub>


’ = ( m - 1)2 - m2 = -2m + 1


Phương trình có nghiệm



’  0  - 2m + 1  0  m  <sub>2</sub>1


- Theo hệ thức Vi- ét, ta có:
x1 + x2 = - <i><sub>a</sub></i>


<i>b</i>


= -2( m - 1) ; x1 . x2 = <i><sub>a</sub></i>
<i>c</i>


=
m2


Bài 31 ( SGK) Tính nhẩm nghiệm của các
phương trình:


b, 3x2 - ( 1 - 3) x - 1 = 0


d, ( m - 1) x2<sub> - ( 2m + 3) x + m + 4 = 0 với</sub>


m  1.


Giải.


b, Ta có : a - b + c = 3 +1 - 3- 1 = 0


Phương trình có hai nghiệm
x1 = -1; x2 = <i><sub>a</sub></i>



<i>c</i>


= <sub>3</sub>3


3
1



d, Ta có : a + b + c


= m - 1 - 2m - 3 + m + 4 = 0
Phương trình có hai nghiệm
x1 = 1; x2 = <i><sub>a</sub></i>


<i>c</i>


= <i>m<sub>m</sub></i><sub></sub><sub>1</sub>4


Bài 32 ( SGK) Tìm hai số u và v trong
mỗi trường hợp sau:


b, u + v = - 42; u.v = - 400
c, u - v = 5; uv = 24


Giải.


b, Ta có: S = u + v = - 42
P = u. v = - 400


 u và v là nghiệm của phương trình



x2<sub> + 42x - 400 = 0</sub>


’ = 212 - (- 400) = 841   =29


x1 = -21 + 29 = 8


</div>
<span class='text_page_counter'>(32)</span><div class='page_container' data-page=32>

GV: Với câu c làm thế nào để đưa về
dạng câu b?


HS làm bài tập 33


GV: Bài toán yêu cầu làm gì?


GV hướng dẫn HS chứng minh.


- áp dụng kết luận trên hãy phân tích đa
thức sau thành nhân tử.


x2 = -21 - 29 = -50


Vậy u = 8; v = -50 hoặc v = 8; u = -50.
c, Có S = u + (-v) = 5, P = u. (-v) = -24


 u và ( -v) là nghiệm của phương trình


x2<sub> - 5x - 24 = 0</sub>


 =25 + 96 = 121   = 11
x1 = <sub>2</sub> 8



11
5





; x2 = <sub>2</sub> 3


11
5






Vậy u = 8; -v = -3  u = 8; v = 3


hoặc u = -3; - v = 8  u = -3, v = -8


Bài 33. Chứng tỏ rằng nếu phương trình
ax2<sub> + bx + c = 0 có nghiệm là x</sub>


1 và x2 thì


tam thức ax2<sub> + bx + c phân tích thành </sub>


nhân tử như sau:
ax2<sub> + bx + c = a(x - x</sub>



1) ( x - x2)


Chứng minh


Ta có : ax2<sub> + bx + c = a(x</sub>2<sub> + </sub>
<i>a</i>
<i>b</i>


x + <i><sub>a</sub>c</i> )
= a  x2 - ( - <i><sub>a</sub>b</i> ) x +<i><sub>a</sub>c</i> 


= a x2 - (x1+ x2) x + x1x2)


= a  (x2 - x1x) - (x2x - x1x2)


= a ( x- x1) (x - x2)


Vậy ax2<sub> + bx + c = a(x - x</sub>


1) ( x - x2)


áp dụng : 2x2<sub> - 5x + 3 = 0</sub>


có a + b + c = 0 ; x1 = 1; x2 = <i><sub>a</sub></i>
<i>c</i>


= <sub>2</sub>3
2x2<sub> - 5x + 3 = 2( x - 1) ( x - </sub>


2


3


)
= (x - 1) ( 2x - 3)


<b>Hoạt động 3 </b> <b>Hướng dẫn về nhà</b> (2’)
- BTVN : 30a, 31 a, c,; 32 a, 33b.


- Đọc trước bài 7. Phương trình quy về phương trình bậc hai


<i>Ngày soạn:</i>

<sub>07/03/2010</sub>

<i>Ngày dạy:</i>

<sub>08/03/2010</sub>



Tiết: 58 <b>§7</b><i><b>. </b></i><b>PHƯƠNG TRÌNH QUY VỀ PHƯƠNG TRÌNH BẬC HAI</b>


I. <b>Mục tiêu</b>


</div>
<span class='text_page_counter'>(33)</span><div class='page_container' data-page=33>

- HS biết cách giải một số dạng phương trình quy về phương trình bậc hai như:
phương trình trùng phương, phương trình có chứa ẩn ở mẫu thức, một vài phương
trình bậc cao có thể đưa về phương trình tích hoặc giải được nhờ ẩn phụ.


- HS ghi nhớ khi giải phương trình chứa ẩn ở mẫu thức trước hết phải tìm điều kiện
của ẩn và phải kiểm tra đối chiếu điều kiện để chọn nghiệm thoả mãn điều kiện đó.
- HS được rèn kĩ năng phân tích đa thức thành nhân tử để giải phương trình tích.
II. <b>Chuẩn bị</b>


GV: Bảng phụ.


HS: Ơn tập cách giải phương trình chứa ẩn ở mẫu thức và phương trình tích.


<b>III. Tiến trình dạy - học</b>


<b>Hoạt động 1</b> (17’)


GV: Ta đã biết cách giải phương trình
bậc hai. Trong thực tế, có những phương
trình khơng phải phương trình bậc hai,
nhưng cũng có thể giải bằng cách quy về
phương trình bậc hai.


Ta xét phương trình trùng phương.
GV giới thiệu phương trình trùng
phương.


HS cho VD


GV: Để giải phương trình trùng phương
ta làm như thế nào?


GV nêu nhận xét.


GV đưa ví dụ 1 lên bảng phụ hướng dẫn
HS từng bước


GV: Tại sao t ≥ 0?
HS làm ?1.


HS hoạt động nhóm
Gọi HS lên bảng giải.


<b>1. Phương trình trùng phương:</b>



* Phương trình trùng phương là phương
trình có dạng ax4<sub>+ bx</sub>2<sub> + c = 0 ( a </sub><sub></sub><sub> 0). </sub>


VD: 5x4<sub> - 16 = 0</sub>


4x4<sub> + x</sub>2<sub> = 0</sub>


x4<sub> + 4x</sub>2<sub> - 15 = 0</sub>


x4<sub> - 5x</sub>2<sub> + 4 = 0</sub>


Nhận xét: Để giải phương trình
ax4<sub> + bx</sub>2<sub> + c = 0 (a </sub><sub></sub><sub> 0) </sub>


ta đặt ẩn phụ x2<sub> = t (t </sub><sub></sub><sub> 0). </sub>


Ta được PT bậc hai at2<sub> + bt + c = 0.</sub>


?1. Giải các phương trình trùng phương.
a, 4x4<sub> + x</sub>2<sub> - 5 = 0 </sub>


Đặt x2<sub> = t , ĐK t </sub><sub></sub><sub> 0. </sub>


4t2<sub> + t -5 = 0 </sub>


Có a + b + c = 4 +1 - 5 = 0


 t1 = 1 ( TM) ; t2 = <sub>4</sub>


5





( loại)
t1 = x2 = 1  x1,2 =  1.


b, Đặt x2<sub> = t </sub><sub></sub><sub> 0</sub>


3t2<sub> + 4t + 1 = 0 </sub>


Có a - b + c = 3 - 4 + 1 = 0


</div>
<span class='text_page_counter'>(34)</span><div class='page_container' data-page=34>

GV bổ sung câu c) x4<sub> - 9x</sub>2<sub> = 0</sub>


GV: Em có nhận xét gì về số nghiệm của
phương trình trùng phương?


 t1 = -1 ( loại ); t2 = -<sub>3</sub>


1


( loại)
Phương trình vơ nghiệm


c, x4<sub> - 9x</sub>2<sub> = 0</sub>


Đặt x2<sub> = t. Điều kiện t </sub>
 0


t2<sub> - 9t = 0 </sub>



 t( t - 9 ) = 0
 t = 0 hoặc t = 9


t1 = x2 = 0  x1 = 0


t2 = x2 = 9  x2,3 =  3.


Nhận xét : Phương trình trùng phương có
thể vơ nghiệm, 1 nghiệm, 2 nghiệm, 3
nghiệm, tối đa là 4 nghiệm.


<b>Hoạt động 2</b> (15’)


GV: Nhắc lại các bước giải phương trình
chứa ẩn ở mẫu thức đã học ở lớp 8?
GV đưa các bước giải lên bảng phụ


HS làm ?2 (bảng phụ)
Giải phương trình


2
2


x 3x 6


x 9


 



 = 3


1




<i>x</i> (3)


GV: Hãy tìm điều kiện xác định của
phương trình?


HS làm bài 35b - SGK
b,


5
2





<i>x</i>
<i>x</i>


+ 3 =


<i>x</i>




2


6


Gọi HS lên bảng giải.


GV: Khi giải phương trình có chứa ẩn ở
mẫu cần lưu ý các bước nào?


<b>2. Phương trình chứa ẩn ở mẫu.</b>


Giải phương trình chứa ẩn ở mẫu thức:
- Tìm điều kiện xác định của phương
trình.


- Quy đồng mẫu thức hai vế rồi khử mẫu
thức.


- Giải phương trình vừa nhận được.
- Trong các giá trị tìm được của ẩn, loại
các giá trị không thoả mãn điều kiện xác
định, các giá trị thoả mãn điều kiện là
nghiệm của phương trình.


?2. Giải phương trình
Giải. Điều kiện x   3.


(3)  x2 - 3x + 6 = x + 3
 x2 - 4x + 3 = 0


Có a + b + c = 1 - 4 + 3 = 0



 x1 = 1 ( TMĐK); x2 = 3 ( loại)


Vậy nghiệm của phương trình (3)là x = 1.
Bài 35 . Giải phương trình


Điều kiện x  5; x  2.


(x +2) ( 2 - x) + 3( x-5) (2-x) = 6(x - 5)


 4 - x2 + 6x -3x2 - 30 + 15x = 6x - 30
 4x2 - 15x - 4 = 0


 = (-15)2 - 4. 4. (-4) = 289  = 17
x1 = 8


17
15


= 4 ( TM);
x2 = 8


17
15


= - 4
1


( TM)


Vậy nghiệm của phương trình là x1 = 4;



x2 = - <sub>4</sub>


1


<b>Hoạt động 3</b> (12’) <b>3. Phương trình tích.</b>


</div>
<span class='text_page_counter'>(35)</span><div class='page_container' data-page=35>

GV nêu bài tốn , u cầu HS giải
phương trình.


GV: Một tích bằng 0 khi nào?


HS làm ?3.


GV: Đưa phương trình x3<sub> + 3x</sub>2<sub> + 2x = 0 </sub>


về phương trình tích bằng cách nào?


Ví dụ 2. Giải phương trình
(x + 1) ( x2<sub>+ 2x - 3) = 0</sub>


Giải. (x + 1) ( x2<sub>+ 2x - 3) = 0</sub>
 x + 1 = 0 hoặc x2+ 2x – 3 = 0


* x + 1 = 0  x1 = -1


* x2<sub>+ 2x - 3 = 0 </sub><sub></sub><sub> x</sub>


2= 1 ; x3 = 2



Vậy nghiệm của phươngn trình là x1= -2;


x2 = 1; x3 = -3.


?3. Giải phương trình sau bằng cách đưa
về phương trình tích. x3<sub> + 3x</sub>2<sub> + 2x = 0</sub>
 x(x2 + 3x + 2) =0


 12


x 0


x 3x 2 0






  




Giải phương trình: x2<sub> + 3x + 2 = 0</sub>


có a – b + c = 1 – 3 + 2 = 0


 x2 = -1 ; x3 = -2


Vậy các nghiệm phương trình là:
x1 = 0, x2 = -1 ; x3 = -2



<b>Hoạt động 4</b> <b>Hướng dẫn về nhà</b> (1’)
- Nắm vững cách giải từng loại phương trình.
- BTVN : 34, 35, 36 ( SGK); 45, 46, 48 (SBT)


<i>Ngày soạn:</i>

<sub>09/03/2010</sub>

<i>Ngày dạy:</i>

<sub>10/03/2010</sub>



Tiết: 59 <b>LUYỆN TẬP</b>


I. <b>Mục tiêu</b>


- Rèn luyện cho HS kĩ năng giải một số dạng phương trình quy được về phương trình
bậc hai: phương trình trùng phương, phương trình chứa ẩn ở mẫu, một số dạng


phương trình bậc cao.


- Hướng dẫn HS giải phương trình bằng cách đặt ẩn phụ.
II. <b>Chuẩn bị</b>


</div>
<span class='text_page_counter'>(36)</span><div class='page_container' data-page=36>

GV: MTBT.


HS: Ôn tập cách giải các loại phương trình, MTBT.


<b>III. Tiến trình dạy - học</b>
<b>Hoạt động 1</b> (8’)


HS1: Giải phương trình x4<sub> - 5x</sub>2<sub> + 4 = 0 </sub>


HS2: Giải phương trình



1
4




<i>x</i> = ( 1)( 2)
2
2





<i>x</i>
<i>x</i>
<i>x</i>
<i>x</i>

<b>Kiểm tra:</b>


HS1: Giải phương trình x4<sub> - 5x</sub>2<sub> + 4 = 0 </sub>


Đặt x2<sub> = t </sub>
 0


Ta có: t2<sub> - 5t + 4 = 0 </sub>


Có a + b + c = = 1- 5 + 4 = 0
 t1 =1; t2 = 4.



t = t1 = 1  x2 = 1  x1 = -1; x2 = 1


t = t2 = 4  x2 = 4  x3 = -2; x4 = 2


HS2: Giải phương trình


1
4




<i>x</i> = ( 1)( 2)
2
2





<i>x</i>
<i>x</i>
<i>x</i>
<i>x</i>


( 1) ĐK: x -1; x 


-2 (1) => 4( x + 2) = - x2<sub> - x + 2</sub>
 4x + 8 = -x2 - x + 2
 x2 + 5x + 6 = 0



 x1 = -2( loại ); x2 = -3


Vậy phương trình có 1 nghiệm x = -3.


<b>Hoạt động 2</b> (35’)


HS làm bài tập 37 ( SGK)
Gọi 2 HS lên bảng giải.


GV: Đối với câu d trước tiên ta phải làm
gì?


HS làm bài tập 38 ( SGK)


GV: Để giải câu a trước tiên ta phải làm
gì?


<b>Luyện tập:</b>


Bài 37 ( SGK) Giải phương trình trùng
phương


c, 0,3x4<sub> + 1,8x</sub>2<sub> + 1,5 = 0 </sub>


Đặt x2<sub> = t </sub>
 0


Ta có : 0,3t2<sub> + 1,8t + 1,5 = 0</sub>


Có a - b + c = 0,3 - 1,8 + 1,5 = 0



 t1 = -1( loại) ; t2 = -1,5 ( loại).


Vậy phương trình đã cho vơ nghiệm.
d, 2x2<sub> + 1 = </sub>


2


1


<i>x</i> - 4 . ĐK : x  0
2x4<sub> + 5x</sub>2<sub> - 1 = 0 Đặt x</sub>2<sub> = t </sub>


 0


2t2<sub> + 5t - 1 = 0 </sub>


 = 25 + 8 = 33
 t1 =


4
33
5


 <sub> ( TMĐK); </sub>


t2 =
4


33


5


 <sub> < 0 ( loại) </sub>
t1 =


4
33
5


 <sub> </sub>


 x1,2 = 


2
33
5


Bài 38 ( SGK) Giải các phương trình
b, x3<sub> + 2x</sub>2<sub> - (x - 3)</sub>2<sub> = ( x - 1) ( x</sub>2<sub> - 2)</sub>
 x3 + 2x2 - x2 + 6x - 9 = x3 - 2x - x2 + 2
 2x2 + 8x - 11 = 0


’ = 16 + 22 = 38 x1, 2 =
2


38
4



</div>
<span class='text_page_counter'>(37)</span><div class='page_container' data-page=37>

GV: Có nhận xét gì về câu d?


GV: Biến đổi phương trình này về dạng
phương trình bậc hai bằng cách nào?


HS làm bài 39-SGK


GV: Đối với câu d ta biến đổi thế nào?


HS làm bài 40(SGK)


GV: Bài toán yêu cầu làm gì?


GV: Để giải phương trình ta đặt ẩn phụ
như thế nào?


d, <i>x</i>(<i>x</i><sub>3</sub> 7) - 1 = <sub>2</sub><i>x</i> - <i>x</i><sub>3</sub> 4


 2x( x - 7) - 6 = 3x - 2( x - 4)
 2x2 - 14x - 6 - 3x + 2x - 8 = 0
 2x2 - 15x - 14 = 0


 = 152 - 4.2.(-14) = 225 + 337,


 = 337


 x1, 2 =
4


337


15


Bài 39 ( SGK) Giải phương trình bằng
cách đưa về phương trình tích.


a,(3x2<sub> - 7x - 10)</sub><sub></sub><sub>2x</sub>2<sub> +(1 -</sub> <sub>5</sub><sub>)x+</sub> <sub>5</sub><sub>-3</sub><sub></sub><sub> = </sub>


0


 3x2 - 7x - 10 = 0


hoặc 2x2<sub> + ( 1 - </sub> <sub>5</sub><sub>)x +</sub> <sub>5</sub><sub> - 3 = 0 </sub>


* 3x2<sub> - 7x - 10 = 0 </sub>


Có a - b + c = 3 + 7 -10 = 0
 x1 = -1; x2 = <sub>3</sub>


10


* 2x2<sub> + ( 1 - </sub> <sub>5</sub><sub>)x +</sub> <sub>5</sub><sub> - 3 = 0 </sub>


Có a + b + c = 2+ 1 - 5 + 5 - 3 = 0
 x3 = 1; x4 =


2
3
5


Vậy phương trình có 4 nghiệm


x1 = -1; x2 =


3
10


; x3 = 1; x4 =
2


3
5 
d, (x2<sub> + 2x - 5)</sub>2<sub> = (x</sub>2<sub> - x + 5)</sub>2
 (x2 + 2x - 5)2 - ( x2 - x + 5)2 = 0


(x2+2x- 5 + x2- x + 5)(x2 +2x -5 -x2 +x-


5)=0


 ( 2x2 + x) ( 3x - 10) = 0
 2x2 + x = 0 hoặc 3x - 10 = 0


* 2x2<sub> + x = 0 </sub>


 x(2x + 1) = 0
 x1= 0; x2= - <sub>2</sub>


1


* 3x - 10 = 0  x3 = <sub>3</sub>


10



Vậy phương trình có 3 nghiệm
x = 0; x2= - <sub>2</sub>


1


; x3 = <sub>3</sub>


10


.


Bài 40 ( SGK) Giải phương trình bằng
cách đặt ẩn phụ.


a, 3(x2<sub> + x)</sub>2<sub> - 2(x</sub>2<sub> + x) - 1 = 0</sub>


Đặt x2<sub> + x = t </sub>


Ta có phương trình 3t2<sub> - 2t - 1 = 0</sub>


Có a + b + c = 3 - 2- 1 = 0


 t1 = 1; t2 = <sub>3</sub>


1




</div>
<span class='text_page_counter'>(38)</span><div class='page_container' data-page=38>

GV: Khi giải phương trình quy về


phương trình bậc hai như khi đặt ẩn phụ
cần chú ý đến điều kiện của ẩn phụ; với
phương trình có chứa ẩn ở mẫu phải đặt
điều kiện cho tất cả các mẫu khác 0. khi
nhận nghiệm phải đối chiếu điều kiện.


với t1 = 1  x2 + x = 1  x2 + x - 1 = 0
 = 1+ 4 = 5  x1, 2 =


2
5
1


Với t = t2 = <sub>3</sub>


1




 x2 + x = <sub>3</sub>1


 3x2 + 3x + 1 = 0  = 9 - 12 = -3 < 0


Phương trình vơ nghiệm.


Vậy phương trình có hai nghiệm là :
x1, 2 =


2


5
1


<b>Hoạt động 3 </b> <b>Hướng dẫn về nhà</b> (2’)
- BTVN : 37a, b ; 38a,c ; 39b,c; 40b,c,d


- Ơn lại các bước giải bài tốn bằng cách lập phương trình.


<i>Ngày soạn:</i>

<sub>14/03/2010</sub>

<i>Ngày dạy:</i>

<sub>15/03/2010</sub>



Tiết: 60 <b>§</b><i><b>8. </b></i><b>GIẢI BÀI TỐN BẰNG CÁCH LẬP PHƯƠNG TRÌNH</b>


I. <b>Mục tiêu</b>


- HS biết chọn ẩn, điều kiện cho ẩn.


- HS biết phân tích mối quan hệ giữa các đại lượng để lập phương trình bài tốn.
- HS biết trình bày bài giải của một bài tốn bậc hai.


II. <b>Chuẩn bị</b>


GV : Bảng phụ , thước thẳng, MTBT.


HS: Ôn tập các bước giải bài toán bằng cách lập phương trình. MTBT.


<b>III. Tiến trình dạy - học</b>
<b>Hoạt động 1</b> (20’)


GV: Để giải bài toán bằng cách lập


phương trình ta làm những bước nào?
GV: Đưa các bước giải lên bảng phụ


<b>Ví dụ:</b>


Để giải bài tốn bằng cách lập phương
trình :


- Bước 1: Lập phương trình.


- Chọn ẩn số, đặt điều kiện thích hợp cho
ẩn.


- Biểu diễn các đại lượng chưa biết theo


</div>
<span class='text_page_counter'>(39)</span><div class='page_container' data-page=39>

GV nêu VD
HS đọc đề bài.


GV: Bài toán cho biết gì?


GV: Bài tốn có những đại lượng nào?
HS: Số áo may trong một ngày, thời gian
may, số áo.


GV: Số liệu nào đã biết? chọn gì làm ẩn?
GV đưa bảng phân tích các đại lượng
cùng hướng dẫn HS điền các đại lượng để
lập phương trình.


Số áo


may 1


ngày


Số ngày Số áo
may
Kế


hoạch


x


( áo) <i>x</i>


3000


(ngày)


3000
(áo)
Thực


hiện


x + 6


( áo) 6


2650





<i>x</i> (ngày


)


2650
(áo)
HS đứng tại chỗ trình bày bài giải.
GV ghi bảng.


ẩn và các đại lượng đã biết.


- Lập phương trình biểu thị mối quan hệ
giữa các đại lượng.


Bước 2: Giải phương trình.


Bước 3: Đối chiếu điều kiện. Trả lời bài
tốn.


Ví dụ
Cho biết :


Kế hoạch may xong 3000 áo.


Thực hiện 1 ngày may nhiều hơn 6 áo so
với kế hoạch và sớm hơn 5 ngày.


Hỏi, theo kế hoạch mỗi ngày may được


bao nhiêu áo?


Giải.


Gọi số áo phải may trong 1 ngày theo kế
hoạch là x ( x nguyên dương)


Thời gian may xong 3000 áo theo kế
hoạch là 3000<i><sub>x</sub></i> (ngày)


Số áo thực tế may trong một ngày là
x + 6 ( áo).


Thời gian may xong 2650 áo là2650<sub>6</sub>




<i>x</i>


(ngày)


Vì xưởng may xong 2650 áo trước khi hết
hạn 5 ngày nên ta có phương trình :


3000<i><sub>x</sub></i> - 5 = 2650<sub>6</sub>




<i>x</i>



 3000 ( x + 6) - 5x( x+ 6) = 2650 x
 3000x+ 18 000- 5x2- 30x- 2650x= 0
 -5x2 - 320x + 18 000 = 0


 x2 + 64x - 3600 = 0


’ = 322 + 3600 = 4624   = 68
 x1 = 32+ 68 = 100 ( TMĐK)


</div>
<span class='text_page_counter'>(40)</span><div class='page_container' data-page=40>

GV: Hãy chọn kết quả và trả lời.?
GV: Nêu lại các bước giải bài toán .
GV: So với bài toán giải bài tốn bằng
cách lập phương trình đã học có gì giống
và khác?


GV nhấn mạnh các bước giải.
HS thảo luận nhóm ?1. ( 5’)


Gọi đại diện 1 nhóm lên bảng giải.


x2 = 32 - 68 = -36 ( loại)


Vậy theo kế hoạch, mỗi ngày xưởng phải
may xong 100 áo.


?1. Giải.


Gọi chiều dài của mảnh vườn là x (m)
ĐK : x > 4



Chiều rộng của mảnh đất là x - 4 ( m)
Theo bài ra diện tích của mảnh đất là
320m2<sub>, ta có phương trình :</sub>


x( x - 4) = 320


 x2 - 4x - 320 = 0


’ = 4 + 320 = 324,  = 18


x1 = 2 + 18 = 20 ( TMĐK)


x2 = 2 - 18 = -16 ( loại)


Vậy chiều dài mảnh đất là 20 m


chiều rộng mảnh đất là 20 - 4 = 16(m)<i><b> </b></i>


<b>Hoạt động 2</b> (23)


HS làm bài tập 42 (SGK)
HS đọc đề bài.


GV: Bài toán cho biết điều gì? yêu cầu
tìm gì?


GV: Chọn ẩn và đặt điều kiện cho ẩn?
GV: Sau một năm tiền lãi tính như thế
nào?



GV: Sau một năm cả vốn lẫn lãi là bao
nhiêu?


GV: Sau hai năm cả vốn lẫn lãi là bao
nhiêu?


GV: Theo bài ra Bác Thời phải trả tất cả
là bao nhiêu ? Hãy tìm mối quan hệ để
lập phương trình?


GV: Giải phương trình tìm nghiệm?


GV: Vậy ta kết luận điều gì?
HS làm bài 43 (SGK)


GV: Bài tốn có mấy đại lượng? Các đại


<b>Luyện tập:</b>


Bài 42 ( SGK)
Giải.


Gọi lãi suất cho vay một năm là x%
ĐK : x > 0 .


Sau một năm cả vốn lẫn lãi là :
2 000 000 + 2 000 000. x%
= 2 000 000 + 20 000 .x
= 20 000 ( 100 + x)



Sau năm thứ hai, cả vốn lẫn lãi là :


20 000 (100 + x) + 20 000 (100 + x ).x%
= 20 000( 100 + x) + 200 ( 100 + x ) x
= 200 ( 100 + x)2


Sau năm thứ hai Bác thời phải trả tất cả
2 420 000 đ; ta có phương trình


200 ( 100 + x)2<sub> = 2 420 000 </sub>
 ( 100 + x)2 = 12 100


 100 + x  = 110


* 100 + x = 110  x1 = 10 ( TMĐK)


* 100 + x = - 110  x2 = - 210 ( loại)


Vậy lãi suất cho vay hàng năm là 10%
Bài 43 (SGK)


</div>
<span class='text_page_counter'>(41)</span><div class='page_container' data-page=41>

lượng liên quan với nhau như thế nào?
GV:Số liệu nào đã biết? Số liệu nào
chưa?


GV đưa bảng phân tích lên bảng phụ


S v t


Đi 120 x 120



x


Về 125 x-5 125


x 5


Gv: Điều kiện của ẩn?


Gọi x(km/h) là vận tốc xuồng lúc đi (x>5)
Vận tốc lúc về là: x – 5 (km/h)


Thời gian đi là: 120


x ( h)


Thời gian về là: 125


x 5 (h)


Theo bài ra ta có phương trình:
120


x + 1 =
125
x 5


 x2 – 5x + 120x – 600 = 125x
 x2 – 10x – 600 = 0



x1 = 30; x2 = - 20 (loại)


Vậy vận tốc của xuồng lúc đi là: 30 km/h


<b>Hoạt động 3 </b> <b>Hướng dẫn về nhà</b> (2’)


- Học nắm chắc cách giải bài tốn bằng cách lập phương trình : cần phải chú ý dạng
bài, xác định được mối quan hệ giữa các đại lượng để lập phương trình và chú ý điều
kiện của ẩn để chọn kết quả thích hợp.


- BTVN : 41,43,44,45 ( SGK); 52, 54, 55, 56 (SBT)


<i>Ngày soạn:</i>

<sub>16/03/2010</sub>

<i>Ngày </i>


<i>dạy:</i>


17/03/2010



Tiết: 61 <b>LUYỆN TẬP</b>


I. <b>Mục tiêu</b>


- HS được rèn luyện kĩ năng giải bài tốn bằng cách lập phương trình qua bước phân
tích đề bài, tìm ra mối liên hệ giữa các dự kiện giữa các dự kiện trong bài để lập
phương trình.


- HS biết trình bày bài giải của một bài toán bậc hai.
II. <b>Chuẩn bị</b>


GV: Thước thẳng, MTBT.


HS : Thước kẻ, MTBT<i><b>.</b></i>


<b>III. Tiến trình dạy - học</b>
<b>Hoạt động 1</b> (8’)


HS1: Chữa bài tập 41 ( SGK)


<b>Kiểm tra:</b>


Giải. Gọi số nhỏ là x
Số lớn là x + 5


Tích của hai số bằng 150 nên ta có
phương trình: x(x+5) = 150


</div>
<span class='text_page_counter'>(42)</span><div class='page_container' data-page=42>

x2<sub> + 5x - 150 = 0 </sub>
 = 52 - 4. 1. (-150 ) = 625
  = 25


x1 =


2
25
5


=10 , x2 =
2



25
5


= -15
Vậy nếu một bạn chọn số 10 thì bạn kia
phải chọn số 15


Nếu một bạn chọn số -15 thì bạn kia
chọn số -10.


<b>Hoạt động 2 (36’)</b>


HS làm bài tập 46 ( SGK)


GV: Bài tốn cho biết gì? u cầu tìm gì?


GV: Em hiểu tính kích thước của mảnh đất
là gì?


HS: Tính kích thước của mảnh đất tức là
tính chiều dài và chiều rộng của mảnh đất.
GV: Chọn ẩn số? đơn vị ? điều kiện?
GV: Biểu thị các đại lượng khác và lập
phương trình bài tốn.


Gọi 1 HS lên bảng giải phương trình.


<b>Luyện tập:</b>



Bài 46 ( SGK)
Tóm tắt


Mảnh đất hình chữ nhật diện tích : 240
m2


Tăng chiều rộng 3 m, giảm chiều dài
4m diện tích khơng đổi.


Tính kích thước của HCN?


Giải.


Gọi chiều rộng của hình chữ nhật là x
(m)


ĐK : x > 0


Vì diện tích mảnh đất là 240 m2<sub> nên </sub>


chiều dài là 240<i><sub>x</sub></i> ( m)


Nếu tăng chiều rộng 3m và giảm chiều
dài


4 m thì diện tích khơng đổi , vậy ta có
phương trình:


( x + 3) (240<i><sub>x</sub></i> - 4) = 240



 240 - 4x + 720<i><sub>x</sub></i> - 12 = 240
 240x - 4x2 + 720 - 12x = 240x
 - 4x2 - 12x + 720 = 0


 x2 + 3x - 180 = 0


 = 9 - 4.1.(-180 ) = 729,  =
27


 x1 =


2
27
3


= -15 ( loại)
x2 = <sub>2</sub>


27
3


= 12 ( TMĐK)
Vậy chiều rộng của mảnh đất là 12 m


</div>
<span class='text_page_counter'>(43)</span><div class='page_container' data-page=43>

GV: Vậy kích thước của mảnh đất là bao
nhiêu?


HS làm bài tập 47 ( SGK)



GV: Bài tốn cho biết gì? u cầu làm gì?


GV: Bài tốn thuộc dạng nào? Có mấy đại
lượng tham gia?


GV đưa bảng phân tích lên bảng phụ
V


(km/h)


T
(h)


S
(km)
Bác Hiệp x + 3


3
30




<i>x</i> 30


Cô Liên x


<i>x</i>


30 <sub>30</sub>



GV: Vậy vận tốc của cô Liên là bao nhiêu?
vận tốc của bác Hiệp?


GV hướng dẫn HS bài 52 (SGK)


GV: Bài toán này thuộc dạng tốn gì? Ta


chiều dài mảnh đất là 240<sub>12</sub> = 20
( m)


Bài 47 ( SGK)


Khởi hành cùng một lúc S = 30 km
vận tốc bác Hiệp >vận tốc cô Liên 3
km/h


bác Hiệp đến trước cơ Liên nửa giờ.
Tính vận tốc mỗi người?


Giải.


Gọi vận tốc của cô Liên là x ( km/h)
ĐK : x > 0


Vận tốc của bác Hiệp là x + 3 ( km/h)
Thời gian cô Liên đi hết quãng đường
30 km là 30<i><sub>x</sub></i> ( giờ)


Thời gian bác Hiệp đi hết quãng đường


30 km là : 30<sub>3</sub>




<i>x</i> ( giờ)


Vì Bác Hiệp đến trước cơ Liên nửa giờ
( <sub>2</sub>1 giờ) nên ta có phương trình:


30<i><sub>x</sub></i> - 30<sub>3</sub>




<i>x</i> = 2
1


 60( x + 3) - 60x = x( x + 3)
 60 x + 180 - 60x = x2 + 3x
 x2 + 3x - 180 = 0


 = 9 + 720 = 729  = 27


 x1 = <sub>2</sub>


27
3


= -15 ( loại)
x2 =



2
27
3


= 12 ( TMĐK)
Vậy vận tốc của cô Liên là 12 km/h
vận tốc của bác Hiệp là 15 km/h.
Bài 52(SGK)


Đổi 40' 2
3


 (h)


Gọi vận tốc của Canô khi nước yên
lặng là x (km/h, x > 3)


</div>
<span class='text_page_counter'>(44)</span><div class='page_container' data-page=44>

cần chú ý điều gì?


HS: Vận tốc xi dịng, ngược dịng.
GV: Nếu chọn ẩn là vận tốc của Canơ khi
nước n lặng, các đại lượng cịn lại được
biểu diễn như thế nào?


GV yêu cầu HS về nhà giải phương trình
ĐS: Vận tốc của Canơ là: 12 km/h


Vận tốc xi dịng là: x + 3 (km/h)


Vận tốc ngược dòng là: x – 3 (km/h)
Thời gian đi xi dịng là: 30


x 3 (h)


Thời gian đi ngược dòng là: 30


x 3 (h)


Theo bài ra ta có phương trình:
30 2 30 6


x 3 3 x 3    


<b>Hoạt động 3 </b> <b>Hướng dẫn về nhà</b> (1’)
- Xem lại cách giải các dạng toán.


- BTVN 48, 49, 50, 51, 52,53 ( SGK); 57, 58, 59, 60 (SBT)


<i>Ngày soạn:</i>

<sub>18/04/2010</sub>

<i>Ngày dạy:</i>

<sub>19/04/2010</sub>



Tiết: 62 <b>LUYỆN TẬP</b>


I. <b>Mục tiêu</b>


- HS tiếp tục được rèn luyện kĩ năng giải bài tốn bằng cách lập phương trình qua
bước phân tích đề bài, tìm ra mối liên hệ giữa các dự kiện giữa các dự kiện trong bài
để lập phương trình.


- HS biết trình bày bài giải của một bài toán bậc hai.


II. <b>Chuẩn bị</b>


GV: Thước thẳng, MTBT.
HS : Thước kẻ, MTBT.


<b>III. Tiến trình dạy - học</b>
<b>Hoạt động 1</b> (10’)


HS1: Chữa bài tập 45 ( SGK)


<b>Kiểm tra:</b>


Bài 45 (SGK)


Gọi số bé là x ( *


x N )


Số lớn là x + 1


Tích của chúng: x(x + 1)


Tổng của chúng là: x + x + 1 = 2x + 1
Theo bài ra ta có phương trình:


x(x + 1) – (2x + 1) = 109
 x2 - x – 110 = 0


1 4.110 441



    ,  21


1


1 21


x 11


2




  (TM) ; 2


1 21


x 10


2




  <i><b> (loại)</b></i>


Vậy hai số cần tìm là: 11 và 12


<b>Hoạt động 2</b> (34’)
HS làm bài tập 49.


<b>Luyện tập:</b>



Bài 49 ( SGK)


</div>
<span class='text_page_counter'>(45)</span><div class='page_container' data-page=45>

GV: Bài tốn cho biết gì? u cầu tìm gì?


GV: Xác định dạng bài tốn?bài tốn có
những đại lượng nào?


GV: Với bài toán làm chung, làm riêng
một cơng việc ta cần chú ý điều gì?
HS: Công việc là một đơn vị


Gọi 1 HS lên bảng giải bước lập phương
trình.


Gọi HS2 lên bảng giải phương trình và
trả lời.


* GV : Với dạng tốn làm chung làm
riêng hay tốn về vịi nước chảy, giữa
thời gian hồn thành cơng việc và năng
suất trong một đơn vị thời gian là hai số
nghịch đảo của nhau. Không được lấy
thời gian HTCV của đội I cộng với thời
gian HTCV của đội II bằng thời gian
HTCV của hai đội. Còn năng suất một
ngày của đội I cộng với năng suất một
ngày của đội II bằng năng suất một ngày
của hai đội.



HS làm bài tập 50 SGK


GV: Bài toán cho biết gì? yêu cầu tìm gì?


Hai đội cùng làm xong công việc: 4 ngày
Làm riêng, đội I làm xong trước đội II : 6
ngày.


Tính thời gian mỗi đội làm riêng để hồn
thành cơng việc?


Giải.


Gọi thời gian đội I hồn thành cơng việc
một mình là x ( ngày) ĐK : x > 0


Thời gian đội II hoàn thành cơng việc một
mình là x + 6 ( ngày)


Một ngày đội I làm được : 1<i><sub>x</sub></i> ( CV),
đội hai làm được : 1<sub>6</sub>




<i>x</i> ( CV)


Một ngày cả hai đội làm được 1<sub>4</sub> ( CV)
Theo bài ra ta có phương trình:


1<i><sub>x</sub></i> + 1<sub>6</sub>





<i>x</i> = 4
1


 4( x + 6) + 4x = x( x + 6)
 4x + 24 + 4x = x2 + 6x
 x2 - 2x - 24 = 0


’ = 12 - 1. (-24) = 25,  = 5


 x1 = 6 ( TMĐK); x2 = - 4 ( loại)


Vậy một mình đội I làm trong 6 ngày thì
xong cơng việc.


Một mình đội II làm trong 12 ngày thì
xong cơng việc.


Bài 50 (SGK)


</div>
<span class='text_page_counter'>(46)</span><div class='page_container' data-page=46>

GV: Bài toán thuộc dạng tốn nào?
GV: Ta cần nhớ cơng thức nào?


GV: Hãy giải bài toán trên.


GV: Gọi HS lên bảng giải bước lập
phương trình.



GV: Gọi HS lên bảng giải phương trình.


GV hướng dẫn HS làm bài 51(SGK)
GV: Nồng độ của một chất được tính như
thế nào?


HS: ct


%
dd


m


C .100%


m




GV: Nếu gọi x là lượng nước trong dung
dịch ban đầu thì nồng độ là bao nhiêu?


Gọi x (g/cm3<sub>) là khối lượng riêng của </sub>


miếng kim loại thứ nhất ( x > 1)


Khối lượng riêng của miếng kim loại thứ
hai là: x – 1 (g/cm3<sub>)</sub>


Thể tích của miếng kim loại thứ nhất là:



880


x (cm


3<sub>)</sub>


Thể tích của miếng kim loại thứ hai là:


858
x 1 (cm


3<sub>)</sub>


Theo bài ra ta có phương trình:
880 10 858


x  x 1


 880(x – 1) + 10x(x – 1) = 858x
 880x – 880 + 10x2 – 10x – 858x = 0


 10x2 +12x – 880 = 0
 5x2 + 6x – 440 = 0


’ = 9 + 5. 440 = 2209,  ' 47


x1 = 8,8 (TM) ; x2 = -10(loại)


Vậy, Khối lượng riêng của miếng kim


loại thứ nhất là: 8,8 g/cm3


Khối lượng riêng của miếng kim loại
thứ hai là: 7,8 g/cm3


Bài 51(SGK)


Gọi x (g) là lượng nước trong dung dịch
ban đầu (x > 0)


Nồng độ muối của dung dịch là: 40


x 40


Sau khi thêm 200 (g) nước vào dung dịch




M


D



V





M


V



D






M D.V



</div>
<span class='text_page_counter'>(47)</span><div class='page_container' data-page=47>

GV: Sau khi thêm 200 (g) nước vào dung
dịch thì nồng độ là bao nhiêu?


GV: Theo bài ra ta có phương trình nào?
GV: u cầu HS về nhà giải


thì nồng độ là: 40


x 240


Ta có phương trình:
40 40 10


x 40 x 240 100   


<b>Hoạt động 5 </b> <b>Hướng dẫn về nhà</b> (1’)
- BTVN , 52,53 ( SGK). bài tập SBT


- Chuẩn bị MTBT Casio, tiết sau thực hành máy tính bỏ túi


<i>Ngày soạn:</i>

<sub>18/4/2010</sub>

<i>Ngày dạy:</i>

<sub>23/4/2010</sub>



Tiết: 63 <b>THỰC HÀNH MÁY TÍNH BỎ TÚI</b>


I. <b>Mục tiêu</b>



- Rèn kỉ năng sử dụng máy tính bỏ túi.
- Biết sử dụng MTBT để kiểm tra kết quả
II. <b>Chuẩn bị</b>


GV: MTBT, bảng phụ
HS: MTBT


<b>III. Tiến trình dạy - học</b>


<b>Hoạt động 1</b> (43’) <b>GV : </b>Nêu bài tốn :
Giải các phương trình bậc hai :


a) x2<sub> + 5x + 4 = 0</sub>


có a – b + c = 1 – 5 + 4 = 0


 x1 = -1 ; x2 = - 4


GV đưa bảng phụ cách giải phương trình bằng máy tính Casiơ fx 500MS


MODE MODE 1 > 2 1 = 5 = 4 = =


Kết quả là : x1 = -1 ; x2 = - 4


b) 2x2<sub> – 7x + 3 = 0</sub>


GV : Hãy dùng MTBT Casiơ fx 500MS giải phương trình trên.


MODE MODE 1 > 2 2 = -7 = 3 = =



Kết quả là : x1 = 3 ; x2 = 0,5


Bài tập: Hãy giải các phương trình và kiểm nghiệm bằng máy tính.
a) x2<sub> – 8x + 15 = 0 b) x</sub>2<sub> + 5x + 6 = 0</sub>


</div>
<span class='text_page_counter'>(48)</span><div class='page_container' data-page=48>

c) x 2<sub> + 4x + 4 = 0 d) 4x</sub>2<sub> + 5x + 1 = 0</sub>


GV: Qua việc sử dụng MTBT để giải phương trình, chúng ta cịn có thể dùng MTBT
để kiểm tra kết quả của việc giải phương trình.


<b>Hoạt động 5 </b> <b>Hướng dẫn về nhà</b> (2’)


- Sử dụng thành thạo MTBT để tính tốn và giải phương trình.
- Ơn tập chương IV.


<i>Ngày soạn:</i>

<sub>25/04/2010</sub>

<i>Ngày dạy:</i>

<sub>26/04/2010</sub>



Tiết: 64 <b>ÔN TẬP CHƯƠNG IV (t1)</b>


I. <b>Mục tiêu</b>


* Ơn tập một cách hệ thống lí thuyết của chương:
+ Tính chất và dạng đồ thị của hàm số y = ax2<sub> ( a </sub>


 0)


+ Các cơng thức nghiệm của phương trình bậc hai


+ Hệ thức Viét và vận dụng để tính nhẩm nghiệm phương trình bậc hai. Tìm hai số
biết tổng và tích của chúng.



II. <b>Chuẩn bị</b>


GV: Vẽ sẵn đồ thị hàm số y =2x2<sub>, y = -2x</sub>2<sub> trên bảng phụ , Viết tóm tắt các kiến thức </sub>


cần nhớ lên bảng phụ.
HS: Thước kẻ, MTBT<i><b>.</b></i>


<b>III. Tiến trình dạy - học</b>
<b>Hoạt động 1</b> (25’)


1. Hàm số y = ax2


GV đưa đồ thị hàm số y = 2x2<sub> và y = -2x</sub>2


vẽ sẵn trên bảng phụ , yêu cầu HS trả lời
câu hỏi 1 ( SGK)




<b>Ôn tập lý thuyết :</b>


1. Hàm số y = ax2


a, Nếu a > 0 thì hàm số y = ax2<sub> đồng biến </sub>


khi x > 0, nghịch biến khi x < 0.


Với x = 0 thì hàm số đạt giá trị nhỏ nhất
bằng 0. Khơng có giá trị nào của x để hàm


số đạt giá trị lớn nhất.


Nếu a < 0 thì hàm số đồng biến khi x <
0, nghịch biến khi x> 0.


Với x = 0 thì hàm số đạt giá trị lớn nhất
bằng 0.


Khơng có giá trị nào của x để hàm số đạt
giá trị nhỏ nhất.


b, Đồ thị hàm số y = ax2<sub> ( a </sub><sub></sub><sub> 0) là một </sub>




4


2


-2


-4


y= 2x2


y= - 2x2


y


O x



</div>
<span class='text_page_counter'>(49)</span><div class='page_container' data-page=49>

GV đưa phần tóm tắt các kiến thức cần
nhớ phần 1. Hàm số y = ax2<sub> ( a </sub>


 0)


2. Phương trình bậc hai
ax2<sub>+bx + c = 0 ( a </sub><sub></sub><sub> 0)</sub>


GV: Gọi 2 HS lên bảng viết công thức
nghiệm tổng quát và công thức nghiệm
thu gọn.


GV: Khi nào dùng công thức nghiệm
tổng quát? khi nào dùng cơng thức
nghiệm thu gọn?


GV: Vì sao khi a và c trái dấu thì phương
trình có hai nghiệm phân biệt?


3. Hệ thức Viét và ứng dụng.
GV đưa bảng phụ


Gọi HS lên bảng điền.


đường cong Parabol đỉnh O, nhận trục Oy
làm trục đối xứng.


- Nếu a > 0 thì đồ thị nằm phía trên trục
hồnh, O là điểm thấp nhất của đồ thị.


- Nếu a < 0 thì đồ thị nằm phía dưới trục
hồnh, O là điểm cao nhất của đồ thị.
2. Phương trình bậc hai ax2<sub>+ bx + c = 0 </sub>


(a<sub></sub>0)


 = b2 - 4ac:
  > 0 thì


phương trình có
hai nghiệm phân
biệt:


x1=
<i>a</i>
<i>b</i>


2





 <sub>; x</sub>


2=
<i>a</i>


<i>b</i>


2







  = 0 thì


phương trình có
nghiệm kép
x1= x2 = -


<i>a</i>
<i>b</i>


2 .


 < 0 thì phương


trình vô nghiệm


’ = b’2 - ac
’ > 0 thì


phương trình có
hai nghiệm phân
biệt:


x1 =
<i>a</i>
<i>b</i> 


 ' <sub>; x</sub>


2=
<i>a</i>


<i>b</i> 
 '


 ’ = 0 thì


phương trình có
nghiệm kép
x1= x2 = - <i><sub>a</sub></i>


<i>b</i>


.


’ < 0 thì


phương trình vơ
nghiệm


3. Hệ thức Viét và ứng dụng


Điền vào chỗ trống để được các khẳng
định đúng.


- Nếu x1, x2 là hai nghiệm của phương



trình


ax2<sub> + bx + c = 0 (a </sub>


 0) thì : x1+ x2 = ...;


x1. x2 = ....


- Muốn tìm hai số u và v biết u + v = S,
u.v = P, ta giải phương


trình ...
điều kiện để có u và v là ...
- Nếu a + b +c = 0 thì phương trình ax2<sub> + </sub>


bx + c = 0 ( a  0 ) có hai nghiệm


x1 = ...; x2 = ....


Nếu ...thì phương trình ax2<sub> + </sub>


bx + c = 0 ( a  0 ) có hai nghiệm


x1 = -1; x2 = ....


<b>Hoạt động 2</b> (18’)


HS làm bài tập 54 ( SGK)


<b> Luyện tập:</b>



Bài 54 ( SGK)


</div>
<span class='text_page_counter'>(50)</span><div class='page_container' data-page=50>

HS lên bảng vẽ đồ thị


GV: Tìm hồnh độ điểm M và M’?


GV : Xác định điểm N có cùng hồnh độ
với M và N’ có cùng hồnh độ với M’ ?
HS làm bài tập 55 ( SGK)


HS giải miệng câu a


GV: HS lên bảng vẽ đồ thị
HS: Lên bảng làm câu c.


a, Hoành độ của M là ( - 4) và hồnh độ
của M’ là 4 vì thay y = 4 vào phương
trình hàm số, ta có <sub>4</sub>1 x2<sub> = 4 </sub>


 x2 = 16


 x1,2 =  4


b, Tung độ điểm N và N’ là ( - 4)


- Điểm N có hồnh độ = - 4; Điểm N’ có
hồnh độ bằng 4


Tính y của N và N’


y = - <sub>4</sub>1 . ( - 4)2<sub> = </sub>


-4
1


.42<sub> = -4</sub>


Vì N và N; có cùng tung độ bằng - 4 nên
NN’ // Ox.


Bài 55 Cho phương trình x2<sub> - x + 2 = 0</sub>


a, Giải phương trình


Có a - b + c = 1 + 1 -2 = 0


 x1 = -1; x2 = - <i><sub>a</sub></i>
<i>c</i>


= 2


b, Vẽ đồ thị hai hàm số y = x2<sub> và y= x + 2 </sub>


trên cùng một hệ trục toạ độ.
c, Với x = -1 ta có


y = (-1)2<sub> = -1 + 2 ( = 1)</sub>


Với x = 2, ta có y = 22<sub> = 2 + 2 ( =4)</sub>
 x = -1 và x = 2 thoả mãn phương trình



của cả hai hàm số  x = -1 và x = 2 là


hoành độ giao điểm của hai đồ thị .


<b>Hoạt động 3 </b> <b>Hướng dẫn về nhà</b> (2’)
- Ơn tập kĩ lí thuyết




4


2


-2


-4


-5 5


</div>
<span class='text_page_counter'>(51)</span><div class='page_container' data-page=51>

- BTVN : Làm các phần bài tập còn lại.


<i>Ngày soạn:</i>

<sub>27/04/2010</sub>

<i>Ngày dạy:</i>

<sub>28/04/2010</sub>



Tiết: 65 <b>ÔN TẬP CHƯƠNG IV (t2)</b>


I. <b>Mục tiêu</b>


- Rèn kĩ năng giải phương trình bậc hai, trùng phương, phương trình chứa ẩn ở mẫu,
phương trình tích, giải bài tốn bằng cách lập phương trình.



- Vận dụng hệ thức Vi-et vào giải tốn.
II. <b>Chuẩn bị</b>


GV: Thước kẻ, MTBT.
HS: Thước kẻ, MTBT.


<b>III. Tiến trình dạy - học</b>
<b>Hoạt động 1</b> (9’)


GV gọi 2 HS lên bảng làm bài 56a và
57d


HS1: Làm bài 56a


HS2: Làm bài 57d (HS khá)


<b>Kiểm tra:</b>


Bài 56. Giải phương trình
a, 3x4<sub> - 12x</sub>2 <sub>+ 9 = 0</sub>


Đặt x2<sub> = t </sub>
 0


3t2<sub> - 12 t + 9 = 0</sub>


Có a + b + c = 3 - 12 + 9 = 0


 t1 = 1 ( TMĐK) ; x2 = 3 ( TMĐK)



t1 = x2 = 1  x1,2 =  1


t2 = x2 = 3  x3,4 =  3


Phương trình có 4 nghiệm
Bài 57 d.


1
3


5
,
0





<i>x</i>
<i>x</i>


=


1
9


2
7


2






<i>x</i>
<i>x</i>
ĐK : x  1<sub>3</sub>


 ( x + 0,5) ( 3x - 1) = 7x + 2
 3x2 - x + 1,5x - 0,5 = 7x + 2
 3x2 - 6,5 x - 2,5 = 0


 6x2 - 13 x - 5 = 0


 = 169 + 120 = 289   = 17
x1 = <sub>12</sub>


17
13


= <sub>2</sub>5 ( TMĐK)
x2 = <sub>12</sub>


17
13


= -<sub>3</sub>1 ( loại)


Phương trình có 1 nghiệm x = <sub>2</sub>5



<b>Hoạt động 2</b> (35’)
HS làm bài 62 SGK


GV : Phương trình có nghiệm khi


<b>Luyện tập:</b>


Bài 62 (SGK)


a) Phương trình có nghiệm  ’ ≥ 0


</div>
<span class='text_page_counter'>(52)</span><div class='page_container' data-page=52>

nào ?


GV : Ta biến đổi tổng bình phương
hai nghiệm của phương trình như thế
nào ?


GV yêu cầu HS về nhà tính :
2 2


1 2


14
49


<i>x</i> <i>x</i> 


HS làm bài 64 SGK


GV: Bài tốn cho biết gì ? u cầu


làm gì?


GV: Bài tốn này thuộc dạng tốn
nào?


GV: Ta chọn ẩn cho đại lượng nào?
HS1 lên bảng lập phương trình?


HS2 lên bảng giải phương trình.


 (m – 1)2 + 7m2 > 0 với mọi gioá trị m


Vậy phương trình ln có nghiệm với mọi giá
trị của m


b) Theo Vi-et:


1 2
2
1 2


2( 1)


7
.


7


<i>m</i>



<i>x</i> <i>x</i>


<i>m</i>
<i>x x</i>





 








 <sub></sub>





Ta có:


2 <sub>2</sub>


2 2 2


1 2 1 2 1 2


2( 1)



( ) 2 . 2


7 7


<i>m</i> <i>m</i>


<i>x</i> <i>x</i>  <i>x</i> <i>x</i>  <i>x x</i> <sub></sub>  <sub></sub>  


 


= 4 2 8 4 14 2 18 2 8 4


49 49


<i>m</i>  <i>m</i>  <i>m</i> <i>m</i>  <i>m</i>




Bài 64 ( SGK)


Gọi vân tốc của xe lửa thứ nhất là x(km/h, x > 0).
Vận tốc xe lửa thứ hai là: x + 5 (km/h)


Thời gian xe lửa thứ nhất đi từ Hà Nội đến
chỗ gặp nhau là 450<i><sub>x</sub></i> (giờ)


Thời gian xe lửa thứ hai đi từ Bình Sơn đến
chỗ gặp nhau là 450<sub>5</sub>





<i>x</i> ( giờ)


Vì xe lửa thứ hai đi sau 1 giờ, nghĩa là thời
gian đến chỗ gặp nhau ít hơn xe thứ nhất 1
giờ. Do đó , ta có phương trình


450<i><sub>x</sub></i> - 450<sub>5</sub>




<i>x</i> = 1


 450 ( x + 5) - 450x = x( x + 5)
 450 x + 2250 - 450x = x2 + 5x
 x2 + 5x - 2250 = 0


 = 25 + 9000 = 9025 ,  = 95
x1 = 45 ( TMĐK); x2 = - 50 ( loại)


Vậy vận tốc của xe lửa thứ nhất là 45 km/h
vận tốc của xe lửa thứ hai là 50 km/ h


<b>Hoạt động 3 </b> <b>Hướng dẫn về nhà</b> (1’)


- Ôn tập kĩ lí thuyết và bài tập chuẩn bị kiểm tra chương IV.
- BTVN : Làm các phần bài tập còn lại.


</div>
<span class='text_page_counter'>(53)</span><div class='page_container' data-page=53>

<i>Ngày soạn:</i>

<sub>01/5/2010</sub>

<i>Ngày dạy:</i>

<sub>05/05/2010</sub>




Tiết: 66 <b>KIỂM TRA CHƯƠNG IV</b>


I. <b>Mục tiêu</b>


- Kiểm tra việc nắm kiến thức cơ bản đã học: Quy tắc giải phương trình bậc hai ; hệ
thức Vi- ét và ứng dụng của chúng.


- Đánh giá, phân loại HS , có kế hoạch bồi dưỡng, phụ đạo cho HS.
II. <b>Chuẩn bị</b>


GV : Đề kiểm tra đã phô tô
HS : Ơn kiến thức chương IV


<b>III. Tiến trình dạy - học</b>


<b>Hoạt động 1</b> () Giáo viên phát đề cho HS


<b>A. Bảng ma trận hai chiều.</b>


<b>Kiến thức</b> Nhận biết Thông hiểu Vận dụng <sub>Tổng</sub>


TN TL TN TL TN TL


Phương trình bậc hai 2



5


2




5


Hệ thức Vi - ét 1


1
1



1
Giải bài toán bằng


cách lập phương trình


1

4


1



4


Tổng 3



9


1


1


4


10


<b>B. Đề bài :</b>


1. Cho phương trình: <i><sub>x</sub></i>2 <sub>(2</sub><i><sub>m</sub></i> <sub>1)</sub><i><sub>x m</sub></i>2 <sub>2</sub><i><sub>m</sub></i> <sub>0</sub>


    


a) Giải phương trình với m =  2.


b) Tìm giá trị của m để phương trình nghiệm kép.


c) Tìm m để hai nghiệan x1, x2 của phương trình thoả mãn <i>x</i>12<i>x</i>22 9


2. Hai ơtơ khởi hành cùng một lúc từ A đến B cách nhau 150 km. Xe thứ nhất có vận


</div>
<span class='text_page_counter'>(54)</span><div class='page_container' data-page=54>

tốc nhỏ hơn xe thứ hai 10 km/h nên đến B muộn hơn xe thứ hai 3


4 giờ. Tính vận tốc


mỗi xe?


<b>C. Đáp án – Biểu điể</b>m:


<b>Bài</b> <b>Câu</b> <b>Nội dung</b> <b>Điểm</b>



<b>1</b>


a) Phương trình vơ nghiệm 3 điểm


b) <sub>Phương trình có nghiệm kép </sub>


 1


4


<i>m</i> 2 điểm


c) <sub>ĐK: </sub> 1


4


<i>m</i>


m = 2


1 điểm


<b>2</b> Lập được phương trình


150 150 3


10 4


<i>x</i>  <i>x</i> 



2,5
điểm
Giải phương trình được x1 = 40 (TM); x2 = - 50 (loại) 1 điểm


Trả lời. Vậy vận tốc xe I là 40 km/h
Vận tốc xe II là 50 km/h


0.5
điểm


<b>Tổng điểm</b> 10


điểm


<b>Hoạt động 2 </b> <b>Hướng dẫn về nhà</b> (2’)
- Ôn tập kiến thức Đại số 9


- Tiết sau ôn tập


<i>Ngày soạn:</i>

<sub>21/04/2010</sub>

<i>Ngày dạy:</i>

<sub>22/04/2010</sub>



Tiết: 67 <b>ÔN TẬP CUỐI NĂM (T1)</b>


I. <b>Mục tiêu</b>


- HS được ôn tập các kiến thức về căn bậc hai.


- HS được rèn kỹ năng về rút gọn, biến đổi biểu thức, tính giá trị của biểu thức và một
vài dạng câu hỏi nâng cao trên cơ sở rút gọn biểu thức chứa căn.



- HS được ôn tập các kiến thức về hàm số bậc nhất, hàm số bậc hai.


- HS được rèn luyện thêm kĩ năng giải phương trình, giải hệ phương trình, áp dụng hệ


</div>
<span class='text_page_counter'>(55)</span><div class='page_container' data-page=55>

thức Viét vào việc giải bài tập
II. <b>Chuẩn bị</b>


GV: Bảng phụ.


HS: Ôn tập chương I: Căn bậc hai, căn bậc ba và làm bài tập 1 đến 5.


<b>III. Tiến trình dạy - học</b>
<b>Hoạt động 1</b> (20’)


GV: Trong tập R các số thực, những
số nào có căn bậc hai? những số nào
có căn bậc ba?


- Nêu ví dụ cụ thể với số dương, số
0, số âm.


HS làm bài tập 1 ( 131 - SGK)


GV: <i>A</i> có nghĩa khi nào?


HS làm bài tập 4 ( T132 - SGK)


GV đưa bài tập lên bảng phụ:
Chọn chữ cái trước kết quả đúng
1. Giá trị của biểu thức



<b>Ôn tập lý thuyết + Bài tập trắc nghiệm:</b>


HS: Trong tập R các số thực, các số lớn hơn 0
có căn bậc hai. Mỗi số dương có hai căn bậc
hai là hai số đối nhau. Số 0 có một căn bậc hai
là 0. Số âm khơng có căn bậc hai.


+ Mọi số thực đều có một căn bậc ba. Số
dương có căn bậc ba là số dương, số 0 có căn
bậc ba là số 0, số âm có căn bậc ba là số âm.
Chữa bài tập 1 ( SGK)


Chọn (C) : các mệnh đề I và IV sai
I. (4).(25) = <sub></sub> <sub>4</sub> .  25vô nghĩa


Sai vì  4 và  25vơ nghĩa


IV. 100 = <sub></sub> 10


Sai vì vế trái 100 biểu thị căn bậc hai số học


của 100 không bằng vế phải là  10.


+ <i>A</i> có nghĩa  A  0


+ Chữa bài tập 4 ( SGK)
Chọn (D) 49


Giải thích :



2 <i>x</i> = 3 ĐK : x  0
 2 + <i>x</i> = 9 <sub></sub> <i>x</i> = 7


 x = 49


Bài tập:


1. Chọn D. 3


vì 2 - <sub>(</sub> <sub>3</sub> <sub>2</sub><sub>)</sub>2




</div>
<span class='text_page_counter'>(56)</span><div class='page_container' data-page=56>

2 - <sub>(</sub> <sub>3</sub><sub></sub> <sub>2</sub><sub>)</sub>2 bằng:


A. - 3 B. 4


C. 4 - 3 D. 3


2. Giá trị của biểu thức

2
3
2
3


bằng:



A. - 1 B. 5 - 2 6


C. 5 + 2 6 D. 2


3. Với giá trị nào của x thì


2
1




 <i>x</i> <sub> có </sub>
nghĩa:


A. x > 1 B. x  1


C. x  2 D. x  1


4. Với giá trị nào của x thì


3


<i>x</i>


khơng có nghĩa:


A. x > 0 B. x = 0
C. x < 0 D. với mọi x


= 2 - (2 - 3)



= 3


2. Chọn B. 5 - 2 6



2
3
2
3



= <sub>(</sub> <sub>3</sub>( 3<sub>2</sub><sub>)(</sub> 2<sub>3</sub>)2 <sub>2</sub><sub>)</sub>



=
2
3
6
2
2
3



 <sub> = 5 -2 </sub>


6



3. Chọn D. x  1.
2


1



 <i>x</i>


có nghĩa  1 <sub>2</sub>


 <i>x</i>
 0


 <i>x</i><sub>2</sub> 1 0


 x  1.


4. Chọn C. x < 0


3


<i>x</i>


khơng có nghĩa  <sub>3</sub><i>x</i> < 0


 x < 0.


<b>Hoạt động 3</b> (24’)


Bài 5 ( tr 132 - SGK)


Chứng minh rằng giá trị của biểu
thức sau không phụ thuộc vào biến


<i>x</i>
<i>x</i>
<i>x</i>
<i>x</i>
<i>x</i>
<i>x</i>
<i>x</i>
<i>x</i>
<i>x</i>


<i>x</i> <sub>.</sub> 1


1
2
1


2


2   


















GV: Chứng minh biểu thức trên
không phụ thuộc vào x bằng cách
nào?


GV : Hãy tìm điều kiện để biểu thức
xác định rồi rút gọn biểu thức.


Bài 7 ( tr 148, 149 SBT)


P = <sub></sub>















1
2
2
1
2
<i>x</i>
<i>x</i>
<i>x</i>
<i>x</i>
<i>x</i>
.
2
)
1


( <i><sub>x</sub></i> 2




a, Rút gọn P.


b, Tính P với x = 7 - 4 3


c, Tìm giá trị lớn nhất của P


GV: Gọi HS lên bảng là câu a và b.



<b>Luyện tập:</b>


Giải.


ĐK : x > 0 , x  1


<i>x</i>
<i>x</i>
<i>x</i>
<i>x</i>
<i>x</i>
<i>x</i>
<i>x</i>
<i>x</i>
<i>x</i>
<i>x</i> 1
.
1
2
1
2


2   


















=(2 )(<sub>(</sub> 1<sub>1</sub>)<sub>)</sub>2<sub>.(</sub>( <sub>1</sub>2<sub>)</sub>)( 1)







<i>x</i>
<i>x</i>
<i>x</i>
<i>x</i>
<i>x</i>
<i>x</i>


.(x 1)( x 1)


x


 



= 2 x 2 x x x x 2 x 2


x


      


=


<i>x</i>
<i>x</i>


2 <sub>= 2.</sub>


Vậy với mọi x > 0, x  1 thì giá trị của biểu


thức không phụ thuộc vào biến.
Bài 7 ( SBT)


Giải. ĐK : x  0 , x  1.


P = <sub></sub>















1
2
2
1
2
<i>x</i>
<i>x</i>
<i>x</i>
<i>x</i>
<i>x</i>
.
2
)
1
( <sub></sub> <i><sub>x</sub></i> 2


2


( x 2)( x 1) ( x 2)( x 1)
P


( x 1) ( x 1)


    





 


P = 2 <sub>(</sub> 2<sub>1</sub><sub>)(</sub> <sub>1</sub><sub>)</sub> 2 2











<i>x</i>
<i>x</i>
<i>x</i>
<i>x</i>
<i>x</i>
<i>x</i>
<i>x</i>
<i>x</i>
.
2
)
1
( <sub></sub> <i><sub>x</sub></i> 2


</div>
<span class='text_page_counter'>(57)</span><div class='page_container' data-page=57>

GV: Với câu c ta biến đổi như thế
nào?



GV hướng dẫn HS


P =


2


)
1
.(


2 


 <i>x</i> <i>x</i> <sub> = </sub> <i><sub>x</sub></i><sub>(</sub><sub>1</sub> <i><sub>x</sub></i><sub>)</sub>


 = <i>x</i> - x.


b, Với x = 7 - 4 3= ( 2 - 3)2
 <i>x</i> = <sub>(</sub><sub>2</sub> <sub>3</sub><sub>)</sub>2


 = 2 - 3


P = <i>x</i> - x = 2 - 3- ( 7 - 4 3) = 3 3 - 5


c, P = <i>x</i> - x = - ( x - <i>x</i>)


P = - 














4
1
4
1
2
1
.
2
)


( <i><sub>x</sub></i> 2 <i><sub>x</sub></i>


P = -


2


1 1


x


2 4



 


 


 


 


Vì -


2


1
x


2


 




 


   0 với mọi x  ĐKXĐ.


 P = -


2



1 1


x


2 4


 


 


 


   4


1


 GTLN của P = <sub>4</sub>1
 <i>x</i> =


2
1


 x = <sub>4</sub>1 ( TMĐK)
<b>Hoạt động 3 </b> <b>Hướng dẫn về nhà</b> (1’)


- Tiết sau ôn tập về hàm số bậc nhất, hàm số bậc hai và giải phương trình, hệ phương trình.
- BTVN số 6,7,9,13 ( tr 132, 133 - SGK).


<i>Ngày soạn:</i>

<sub>21/04/2010</sub>

<i>Ngày dạy:</i>

<sub>27/04/2010</sub>




Tiết: 68 <b>ÔN TẬP CUỐI NĂM (t2)</b>


I. <b>Mục tiêu</b>


- HS được ôn tập các kiến thức về hàm số bậc nhất, hàm số bậc hai.


- HS được rèn luyện thêm kĩ năng giải phương trình, giải hệ phương trình, áp dụng hệ
thức Viét vào việc giải bài tập.


- Ôn tập cho HS các bài tập giải tốn bằng cách lập phương trình ( gồm cả giải tốn
bằng cách lập hệ phương trình)


- Tiếp tục rèn luyện cho HS kỹ năng phân loại bài tốn, phân tích các đại lượng của
bài tốn, trình bày bài giải.


- Thấy rõ tính thực tế của toán học.
II. <b>Chuẩn bị</b>


GV : Bảng phụ ghi bài tập.


HS: ôn tập về hàm số bậc nhất, hàm số bậc hai y = ax2<sub> ( a </sub><sub></sub><sub> 0), giải hệ phương trình </sub>


bậc nhất hai ẩn, phương trình bậc hai, hệ thức viét.


<b>III. Tiến trình dạy - học</b>
<b>Hoạt động 1</b> (24’)


GV: Nêu tính chất của hàm số bậc nhất
y = ax + b ( a  0 )



GV: Đồ thị hàm số bậc nhất có dạng như


<b>Ơn tập lý thuyết + Bài tập trắc nghiệm</b>


Hàm số bậc nhất y = ax + b ( a  0) xác


định với mọi x thuộc R và đồng biến trên
R khi a >0, nghịch biến trên R khi a < 0.


- Đồ thị hàm số bậc nhất là một đường


</div>
<span class='text_page_counter'>(58)</span><div class='page_container' data-page=58>

thế nào?


HS làm bài tập trên bảng phụ:


Điểm nào sau đây thuộc đồ thị hàm số
y = -3x + 4


A. ( 0; <sub>3</sub>4 ) B. ( 0; - <sub>3</sub>4 )
C. ( -1; - 7) D. ( -1; 7)
Bài 12 ( SBT)


Điểm M ( -2,5; 0) thuộc đồ thị của hàm
số nào sau đây?


A. y = <sub>5</sub>1 x2<sub> ; B. y = x</sub>2


C. y = 5x2<sub> ; D. không thuộc cả ba đồ </sub>


thị các hàm số trên.


Bài tập bổ sung .


Chọn chữ cái trước kết quả đúng.


1. Phương trình 3x - 2y = 5 có nghiệm là:
A. ( 1; -1) B. ( 5; -5)


C. ( 1; 1) D. ( -5; 5)


2. Hệ phương trình
có nghiệm là:


A. ( 4; - 8); B. ( 3; -2)
C. ( -2; 3) D. ( 2; -3)


3. Cho phương trình 2x2<sub> + 3x + 1 = 0</sub>


Tập nghiệm của phương trình là:
A. ( -1; 1<sub>3</sub>) B. (-<sub>2</sub>1 ; 1)
C. ( -1; -1<sub>2</sub> ) D. ( 1; 1<sub>2</sub> )
4. Phương trình 2x2<sub> - 6x + 5 = 0 có tích </sub>


hai nghiệm bằng:


A. <sub>2</sub>5 B.  5<sub>2</sub>


C. 3 D. không tồn tại
Bài 14 ( SGK) Gọi x1, x2 là hai nghiệm


của phương trình 3x2<sub> - ax - b = 0. Tổng x</sub>


1


+ x2 bằng:


A. -<i>a</i><sub>3</sub>; B. <sub>3</sub><i>a</i> ; C. <sub>3</sub><i>b</i> ; D.


-3


<i>b</i>


Hãy chọn câu trả lời đúng.


thẳng cắt trục tung tại điểm có tung độ
bằng b, song song với đường thẳng y = ax
nếu b  0, trùng với đường thẳng y = ax


nếu b = 0


<b>Bài tập:</b>


Chọn D ( -1,7)


vì thay x = -1 và phương trình y = -3x + 4


Bài 12 ( SBT)
Chọn (D)


Giải thích : cả ba hàm số trên có dạng
y = ax2<sub> ( a </sub>



 0) nên đồ thị đều đi qua gốc


toạ độ, mà không đi qua điểm M( -2,5; 0)


1. Chọn A. ( 1; -1)


Giải thích : thay x = 1; y = -1 vào vế trái
phương trình được


3.1 - 2( -1) = 5


 ( 1; -1) là một nghiệm của phương


trình.


2. Chọn D. ( 2; -3)
Giải thích:


- Cặp số ( 2; -3) thoả mãn cả hai phương
trình của hệ. Hoặc giải hệ phương trình.
3. Chọn C. ( -1; -<sub>2</sub>1 )


Giải thích : Phương trình có
a - b + c = 2 - 3 + 1 = 0


 x1 = - 1; x2 = - <i><sub>a</sub></i>
<i>c</i>


= - 1<sub>2</sub>
4. Chọn ( D) không tồn tại


Giải thích: ’ = 9 - 10 = -1 < 0.


Phương trình vơ nghiệm.
Bài 14 ( SGK) Giải.
Theo hệ thức Vi-ét ta có
x1 + x2 = <sub>3</sub>


<i>a</i>


Vậy chọn B. <sub>3</sub><i>a</i>
Bài 15 ( SGK) Giải




5x + 2y = 4
2x - 3y = 13


</div>
<span class='text_page_counter'>(59)</span><div class='page_container' data-page=59>

Bài 15 ( SGK) Hai phương trình


x2<sub> + ax + 1 = 0 và x</sub>2<sub> - x - a = 0 có một </sub>


nghiệm thực chung khi a bằng :


A. 0; B. 1; C. 2; D. 3.
Hãy chọn câu trả lời đúng.


Nghiệm chung nếu có của hai phương
trình là nghiệm của hệ


x2<sub> + ax + 1 = 0 (1)</sub>



x2<sub> - x - a = 0 ( 2)</sub>


Trừ từng vế (1) và (2), được
( a + 1) ( x +1) = 0
a = -1


x = -1


Với a = -1 thì (1) là x2<sub> - x + 1 = 0 vô </sub>


nghiệm  loại.


Với x = -1 thay vào (1) được
1 - a + 1 = 0  a = 2.


Vậy a = 2 thoả mãn. Chọn (C).


<b>Hoạt động 2</b> (20’)


HS làm bài 7 ( Tr 132 - SGK)
GV : (d1) y = ax + b


( d2) y = a’x + b’


song song với nhau, trùng nhau, cắt nhau
khi nào?


HS: ( d1 )  ( d2) 



(d1) // ( d2) 


( d1) cắt (d2)  a  a’


GV: Gọi 3 HS lên bảng trình bày 3
trường hợp.


HS làm bài 9 ( SGK) Giải hệ phương
trình


GV gợi ý : cần xét hai trường hợp y  0


và y < 0


<b>Luyện tập:</b>


Bài 7 ( SGK)
Giải
a, ( d1 )  ( d2) 




b, ( d1) cắt (d2)  m + 1  2


 m  1.


c, (d1) // ( d2) 


Bài 9 ( SGK) Giải.
a,



Xét trường hợp y  0 y= y


(I)  




 


Xét trường hợp y < 0 y= -y


(I)  




 




a = a’
b 


b’
a = a’
b = b’


m + 1 = 2
5 = n
m = 1
n = 5



m + 1 = 2
5  n


2x + 3  y = 13


3x - y = 3


2x + 3y = 13


9x - 3y = 9 <sub> 3x - y = 3</sub>11x = 22
x = 2


6 - y = 3 <sub> y = 3 ( TM y > 0)</sub>x = 2


2x - 3y = 13
9x - 3y = 9


-7x = 4
<b> 3x - y = 3</b>


x =-


3. (- )- y = 3


x = -


y = - ( TM y < 0)





m = 1
n ≠ 5


</div>
<span class='text_page_counter'>(60)</span><div class='page_container' data-page=60>

Câu b HS về nhà giải.


HD: cần đặt điều kiện cho x, y và giải hệ
phương trình bằng ẩn số phụ.


ĐS : x = 0; y = 1.
HS làm bài 16 ( SGK)


Gọi 2 HS lên bảng giải.


Bài 16 ( SGK) Giải các phương trình
a, 2x3<sub> - x</sub>2<sub> + 3x + 6 = 0</sub>


 2x3 + 2x2 - 3x2 - 3x + 6x + 6 = 0


 2x2 ( x + 1) - 3x( x + 1) + 6( x + 1) = 0
 ( x + 1) ( 2x2 - 3x + 6) = 0


* x + 1 = 0  x = -1


hoặc 2x2<sub> - 3x + 6 = 0</sub>


 = 9 - 48 = - 39 < 0  phương trình vơ


nghiệm



Vậy phương trình đã cho có một nghiệm
x = -1.


<b>Hoạt động 3 </b> <b>Hướng dẫn về nhà</b> (1’)


- Xem lại các dạng toán đã học để ghi nhớ cách phân tích.
- BTVN : các bt còn lại ( SGK), bt 17 ( SBT).


<i>Ngày soạn:</i>

<sub>03/05/2009</sub>

<i>Ngày dạy:</i>

<sub>04/05/2009</sub>



Tiết: 69 <b>ÔN TẬP CUỐI NĂM (t3)</b>


I. <b> Mục tiªu</b>


Ôn tập cho HS các bài tập giải bài toán bằng cách lập pt ( gồm cả giải bài toán
bằng cách lập hệ phương trình )


Tiếp tục rèn luyện cho hs kĩ năng phân tích loại bài tốn , phân tích các đại lượng
của bài tốn , trình bày bài giải .


Thấy rõ tính thực tế của tốn học
II. <b>Chuẩn bị</b>


GV : Bảng phụ
HS : Ơân tập


<b>III. Tiến trình dạy học</b>
<b>Hoạt động 1</b> (15’)


HS1: Chữa bài 12 / 133 sgk



<b>Kiểm tra:</b>


Bài 12 / sgk:


Gọi vận tốc lên dốc của người đó là x
( km / h )


Vận tốc lúc xuống dốc là y ( km / h )
ĐK : 0 < x < y


Khi đi từ A đến B thời gian hết 40’ = 2


3 h


nên ta có phương trình : 4 5<sub>x</sub><sub>y</sub>2<sub>3</sub>


Khi đi từ B về A hết 41 phút = 41


60 h nên


ta có phương trình :


</div>
<span class='text_page_counter'>(61)</span><div class='page_container' data-page=61>

HS2 : Chữa bài 17 / 134


GV và HS nhận xét


<sub>x</sub>54<sub>y</sub><sub>60</sub>41


Ta có hệ phương trình :



4 5 2


x y 3


5 4 41


x y 60




 






  




Giải hệ phương trình ta được: 12


15


x
y










Vậy vận tốc lên dốc là 12 km/h, vận tốc
xuống dốc là 15 km/h


HS2 : Bài 17 / 134 sgk


Gọi số ghế băng lúc đầu có là x ( ghế )
ĐK : x > 2 và x nguyên dương


Thì số HS ngồi trên1ghế lúcđầu là:40


x (hs)


Số ghế sau khi bớt là : ( x – 2 ) ghế
Số HS ngồi trên 1 ghế lúc sau là: 40


x 2 (hs)


Ta có phương trình :
40


x 2 -


40
x = 1



Giải phương trình ta được x = 10 (TM)
Vậy trong phịng có 10 chiếc ghế băng.


<b>Hoạt động 2</b> (28’)
HS làm bài 16/SBT


GV: Ta chọn ẩn cho đại lượng nào?


GV: Diện tích của tam giác được tính
như thế nào?


Y/C một học sinh lên bảng lập phương
trình, một học sinh giải phương trình


<b>Luyện tập:</b>


Bài 16 ( tốn nội dung hình học )
Gọi chiều cao của tam giác là x ( dm )
Cạnh đáy của tam giác là y ( dm )
ĐK : x ; y > 0


Ta có phương trình : x = 3


4y


Nếu chiều cao tăng thêm 3 dm và cạnh
đáy giảm đi 2 dm thì diện tích của nó tăng
thêm 12 dm2<sub> ta có pt : </sub>



(x 3)(y 2) xy


12


2 2


 


 


 xy – 2x + 3y – 6 = xy + 24
 -2x + 3y = 30


Ta có hệ phương trình :


3


x y


4


2x 3y 30







  





3


x y <sub>x 15</sub>


4


3 y 20


2. y 3y 30
4





 <sub></sub> <sub></sub>




 <sub></sub>  <sub></sub>





  






( TM ĐK )
Trả lời :Chiều cao của tam giác là 15 dm


</div>
<span class='text_page_counter'>(62)</span><div class='page_container' data-page=62>

HS làm bài 18/SBT


HS lên bảng trình bày lời giải


Cạnh đáy của tam giác là 20 dm
Bài 18 / 150 SBT ( toán về quan hệ số )
Gọi hai số cần tìm là : x và y


Ta có hệ phương trình :


2 2


x y 20


x y 208


 





 




Từ (1 )  ( x + y )2 = 400



Hay x2<sub> + y</sub>2<sub> + 2xy = 400 </sub>


Mà x2<sub> + y</sub>2<sub> = 208 </sub>


 2xy = 400 – 208 = 192  xy = 96


Vậy xy là hai nghiệm của pt :
X2<sub> – 20 X + 96 = 0 </sub>


Giải pt ta được X1 = 12 ; X2 = 8


Vậy hai số cần tìm là 12 và 8


<b>Hoạt động 3 </b> <b>Hướng dẫn về nhà</b> (2’)


Xem lại các dạng tốn đã học để ghi nhớ cách phân tích


Làm thêm loại toán Làm chung , làn riêng ( Bài 13 phần hướng dẫn ôn tập )


<i>Ngày soạn:</i>

<sub>10/05/2010</sub>

<i>Ngày dạy:</i>

<sub>11/05/2010</sub>



Tiết: 70 <b>TRẢ BÀI KIỂM TRA CUỐI NĂM</b>


I. <b>Mục tiêu</b>


- Giúp HS đánh giá được kết quả của bản thân.
- Luyện tập cách làm bài thi


II. <b>Chuẩn bị</b>



GV : Đề bài + đáp án đề thi


<b>III. Tiến trình dạy - học</b>
<b>Hoạt động 1</b> GV nêu đề bài:


<b>Câu 1: ( 2 điểm) </b>Cho hàm số y = ax + b


a) Xác định hệ số a, b để đồ thị của hàm số trên đi qua điểm A(1;3) và B(-1;7)
b) Điểm 1 2


2


C( ; )<sub> có thuộc đường thẳng chứa đồ thị vừa xác định không? Vì sao?</sub>


<b>Câu 2: ( 2 điểm)</b>


Một sân trường hình chữ nhật có chu vi 440 m. Ba lần chiều dài hơn bốn lần chiều
rộng là 30 m. Tính chiều dài và chiều rộng của sân trường.


<b>Câu 3: ( 3 điểm)</b>


Cho phương trình: 2x2<sub> + (3m – 1)x + 2 = 0</sub>


a) Giải phương trình khi m = 2


b) Tính tổng các bình phương của hai nghiệm vừa tìm được.




(1)


(2)


</div>
<span class='text_page_counter'>(63)</span><div class='page_container' data-page=63>

c) Tìm m đề phương trình đã cho có nghiệm két.


<b>Câu 4: ( 2 điểm)</b>


Trên nửa đường trịn đường kính AD lấy hai điểm B và C ( Bnằm giữa A và C). AC
cắt BD tại E. Kẻ EF  AD tại F. Gọi M là trung điểm của ED. Chứng minh :


a) Các tứ giác ABEF và DCEF nội tiếp.
b) Tia BD là tia phân giác của góc CBF;
c) Tứ giác BCMF nội tiếp.


<b>Hoạt động 2</b> () GV chữa bài tóm tắt.


<b>Câu 1: ( 2 điểm) </b>Cho hàm số y = ax + b


a) Điểm A(1;3) và B(-1;7) thuộc đồ thị hàm số y = ax + b nên ta có:


3
7


a b
a b


 






  


 => a = - 2; b = 5 => y = - 2x + 5


b) Điểm 1 2
2


C( ; ) không thuộc đồ thị của hàm số


<b>Câu 2: ( 2 điểm) </b>


Nữa chu vi hình chữ nhật: 220 m


Gọi x (m) là độ dài chiều dài hình chữ nhật ( x > 0)
y (m) là độ dài chiều rộng hình chữ nhật ( y > 0)
Ta có hệ phương trình: 220


3 4 30


x y
x y


 





 


 Chiều dài: 130 m



Chiều rộng: 90 m


<b>Câu 3: ( 3 điểm)</b>


Cho phương trình: 2x2<sub> + (3m – 1)x + 2 = 0</sub>


a) m = 2 => 2x2<sub> + (3.2 – 1)x + 2 = 0</sub>


 2x2 + 5x + 2 = 0 x1 = -0,5; x2 = - 2


b) Theo Vi – et: x1 + x2 = - 2,5


x1.x2= 1


Ta có: 2 2 2 2


1 2 1 2 2 1 2 2 5 2 1


x x (x x )  x x  ( , )  . = 6,25 – 2 = 4,25


c) Phương trình có nghiệm két   = 0


 (3m – 1)2 – 4.2.2 = 0  9m2 – 6m + 1 – 16 = 0
 9m2 – 6m – 15 = 0 Có a - b + c = 9 + 6 – 15 = 0


=> m1 = - 1 ; m2 = 0,6


<b>Câu 4: ( 3 điểm)</b>



a) <sub>ABE AFE</sub>  <sub>180</sub>o


 


M
F


E


D
A


B


C


</div>
<span class='text_page_counter'>(64)</span><div class='page_container' data-page=64>

<sub>ECD DFE</sub>  <sub>180</sub>o


 


=> Các tứ giác ABEF và DCEF nội tiếp
b) <sub>EBF EBC( CAD</sub> <sub></sub> <sub></sub>


=> Tia BD là tia phân giác của góc CBF


c) <sub>BMF BCF</sub> <sub></sub> <sub> (Đỉnh C và M nhìn cạnh BF dưới hai góc bằng nhau )</sub>


=> Tứ giác BCMF nội tiếp


<b>Hoạt động 3 </b> <b>Hướng dẫn về nhà</b> ()


Ơn lại kiến thức hình học 9


</div>

<!--links-->

Tài liệu bạn tìm kiếm đã sẵn sàng tải về

Tải bản đầy đủ ngay
×